You are on page 1of 40

w

w
w
.
n
e
u
t
r
i
n
o
.
r
o
Societatea de tiine Matematice din Romnia
Filiala Cara-Severin


REVISTA DE
MATEMATIC



A ELEVILOR I PROFESORILOR


DIN JUDEUL
CARA-SEVERIN

Nr. 17 , An VII-2006

Editura Neutrino
Reia, 2006
2

2006, Editura Neutrino
Titlul: Revista de matematic a elevilor i profesorilor din judeul
Cara-Severin
I.S.S.N. 1584-9767




Colectivul de redacie:

Bdescu Ovidiu
Dragomir Adriana
Dragomir Lucian
Didraga Iacob
Gdea Vasilica
Golopena Marius
Moatr Lavinia
Pistril Ion Dumitru
Stniloiu Nicolae
andru Marius
uoi Paul









2006, Editura Neutrino
Toate drepturile rezervate
Mobil: 0724224400
www.neutrino.ro
E-mail: editura@neutrino.ro
w
w
w
.
n
e
u
t
r
i
n
o
.
r
o
3

CUPRINS





Gnduri despre matematic i nu numai pag. 4
Interviu cu Anca Vcrescu ( Lucian Dragomir) .. pag. 5
Chestiuni metodice ,note matematice
Implementarea leciilor AEL n orele de matematic
( Irina Avrmescu,Vasile Chi) pag. 8
Proiect de opional (Dorina Humia,Maria Mirulescu)
. pag.17
Distane ( Nicolae Stniloiu) . pag.22
Funcii periodice ( Mihai Monea ) .. pag.24
Tabra naional Poiana Pinului, 2006 pag.27
Probleme rezolvate pag.28
Concursul revistei ediia a II-a (regulament , probleme
propuse) .. pag.61
Rubrica rezolvitorilor .. pag.75








4

Gnduri despre matematic
i nu numai

n fiecare tiin este numai atta tiin adevrat ct
matematic conine.
Immanuel Kant
Matematica reprezint n sine o colecie de rezultate care pot fi
aplicate la orice.
Bertrand Russell
Geometria este arta de a judeca pe desene ru efectuate.
Niels H. Abel
Dac cineva vrea s determine cu un cuvnt laconic i expresiv
esena matematicii, acela trebuie s spun c este o tiin despre
infinit.
Henri Poincar
Exist zerouri crora li se pare c sunt elipse i n jurul lor se
nvrte toat lumea.
S. E. Lec
Eu am vzut cum odat Laplace a ncercat timp de o or s
restabileasc lanul raionamentelor voalate de ctre el n "Mecanica
cereasc" prin intermediul cuvintelor "este uor de vzut c".
Din amintirile unui elev de al lui Laplace
Cu ct mai mult nvei, cu att mai mult tii.
Cu ct mai mult tii, cu att mai mult uii.
Dac mai mult uii, mai puin tii.
Iar dac mai puin tii, mai puin uii.
Dar dac mai puin uii, mai mult tii.
Atunci pentru ce s nvei?
Din folclorul savanilor

Matematica seamn cu o moar: dac vei turna n ea boabe de
gru, vei obine fin, iar dac vei turna n ea tre, tre i vei
obine.
A. Huxley
w
w
w
.
n
e
u
t
r
i
n
o
.
r
o
5
S ai o via echilibrat,s nvei un pic din toate,s te bucuri de
vrsta pe care o ai
(interviu cu Anca Vcrescu realizat de Lucian Dragomir)

Sperm s v bucurai n cele ce urmeaz mpreun cu noi , pentru
c am reuit dup civa ani o rentlnire de suflet.E vorba de cteva
minute petrecute n compania uneia dintre cele mai premiate eleve din
judeul nostru n cadrul concursurilor de matematic.E vorba de una
dintre cele mai simpatice olimpice pe care am cunoscut-o .E vorba de
Anca Vcrescu,nscut n Caransebe(27 iunie 1982),absolvent a
Liceului Traian Doda ,colecionara unei impresionante galerii de premii
obinute la concursurile de matematic : olimpiada judeean premiul I
( clasele V-XI),premiul II(clasa a XII a ),olimpiada naional premiul I
(clasa aVII a ),premiul II (clasa a XII a ),premiul III (clasele VIII-
XI);dup frumoii ani de liceu,Anca a fost student la Florida
International University din Miami ( 2001-2005 ) , absolvent Summa
Cum Laudae.Din 2005 urmeaz cursuri de doctorat n matematic la
Standford Univerity of California.
Iat acum cte ceva din ce am discutat:
1.Care a fost primul contact cu matematica ?
Cred c am nceput s acord mai mult atenie matematicii dect
celorlalte materii din ciclul primar,atunci cnd doamna nvtoare Florica
Fran ne-a ncurajat s facem probleme suplimentare i ne-a motivat prin
diferite concursuri.
2. Ce profesori i-au marcat drumul prin coal ?
Toi profesorii mei de matematic au avut o contribuie
important n formaia mea intelectual i mai ales academic. Doamna
profesoar Lavinia Moatr, care ne-a prezentat nc din clasa a 5-a
teoreme i formule dificile, mi-a motivat curiozitatea matematic si m-a
nvat s gndesc riguros, abstract. Dorina de a-mi depi propriile
limite, disciplina de a nva ntr-un ritm alert si persistent, toate le-am
dobndit de la acea vrsta fraged datorita doamnei Moatr i s-au
dovedit caliti nepreuite de-a lungulor aniilor. De asemenea rmn
profund recunosctoare domnului profesor Ion Poru, ct i domnilor
profesori Iacob Didraga si Mihail Neacu.
3. Cum te pregteai pentru concursuri,program aproximativ?
Pregtirea mi lua destul de mult timp, de la 4 ore pe zi cnd nu
aveam multe teme la alte materii, pn la 10 ore pe zi n preajma
olimpiadelor.
6
4. Care premiu a fost cel mai muncit , de care te leag amintirile cele
mai puternice?
Nu pot s spun c un anume premiu mi-a adus o satisfacie
deosebit fa de celelalte. M-am pregtit foarte intens n fiecare an, i
dup atta munc i stres, un premiu este numai cireaa de pe tort. Faptul
n sine c ai acceptat provocarea pe care i-o prezint matematica i lupta
cu tine nsi de a nu renuna n faa unor probleme grele este adevarata
rsplata pe care o ai din aceasta experien a olimpiadelor.
5. Vreo amintire deosebit din anii de coal(nu neaprat legat de
matematic)?
Legat de olimpiade totui, mi vine n minte sptmna naionalei
din fiecare an: cltoria pn acolo, locurile noi pe care le vizitezi,
oamenii pe care i cunoti, prietenii pe care i-i faci. Este un avantaj la
care nu m-am gndit la nceput cnd am ales s fac matematic, dar pe
care l-am avut chiar i dup anii olimpiadelor prin posibilitatea de a studia
n SUA i de a merge n viitor, sper, la conferine n diferite ri.
6. Cu cine din ar mai ii legtura?
Cu toata lumea de care am fost apropiat. Internetul face minuni.
7. Ce faci acum , cam cu ce te ocupi ( eventual detalii matematice)?
Tocmai am trecut examenele pentru candidatura la doctorat, care
se dau la Stanford la sfritul anului I. n urmatoarele luni, va trebui s
imi aleg un profesor cu care s lucrez, i s ncep cercetarea pentru teza
de doctorat. Deocamdat m gndesc s studiez geometrie algebric, dar
vreau ca n aceti 5 ani de doctorat s mi iau i cursuri pentru un master
n matematica financiar, pentru a avea mai multe posibiliti de lucru n
viitor. Dup 5 ani n SUA, trebuie s recunosc c am nceput s gndesc
un pic ca americanii, adic practic.
8. Ct de mult simi c te-a ajutat n ceea ce faci acum efortul depus
n anii de gimnaziu i de liceu la matematic?Ct de mult merit s
participe un elev la olimpiade,ct de mult merit s i ocupe timpul
cu "spargerea"unor teme si probleme dificile(n timp ce alii se
relaxeaz)?
Nu cred c cineva i poate garanta c olimpiadele te vor ajuta
concret n viitor, dar cred c nu ai nimic de pierdut,ba chiar numai de
ctigat dac mcar ncerci s excelezi ntr-un anume domeniu. Eu am
continuat pe matematic i cred c participarea la olimpiade m-a ajutat
imens n cariera universitar: raionamentul problemelor de olimpiade l
regsesc de multe ori n probleme dificile la care lucrez acum, iar ritmul
asiduu de lucru parc mi se pare acceptabil dup focul pregtirii pentru
w
w
w
.
n
e
u
t
r
i
n
o
.
r
o
7
concursuri. n plus, participarea repetat la olimpiade arat foarte bine
ntr-un C.V: d dovada unui caracter motivat, perseverent, cruia nu i
este fric s munceasc n plus i n general poate ajuta enorm n procesul
de aplicare pentru o facultate sau un post de lucru.
9.Un ndemn pentru micii olimpici creni ?
Cred c dac nu eti dispus s faci un efort deosebit, nu poi s te
astepi la un rezultat deosebit, i asta nu numai n matematic. n principal
ns, un elev nu ar trebui s munceasca numai cu gndul la rezultate i la
premii, pentru c de multe ori matematica poate deveni foarte frustranta,
fr rezultate i satisfacii imediate, iar premiile sau lipsa de premii nu
este complet reprezentativ pentru potenialul tu matematic. Dar, dac
lucrezi la matematic din pasiune i curiozitate, o s ai numai de ctigat
din strduina ta: o s ai o gndire riguroas, o imaginaie creativ i n
plus o s nvei o mulime de lucruri nu numai folositoare, dar i
interesante. Nu uita ns s ai o via echilibrat, s nvei un pic din toate,
i s te bucuri de vrsta pe care o ai acum. Mult succes!
10. Cstig premiul I la olimpiada judeean de civa ani i
deocamdat la naional nu iau nici un premiu , merit s continui
munca enorm la matematic sau s fac altceva : sport , romna.
fizic,istorie?
O parte din rspuns cred c se regsete n ce spuneam mai
nainte.Voi insista ns i eu :Dac ntr-adevr i place s lucrezi la
matematic, binenelesc merit. De fapt, ar trebui s ncerci cte puin
din orice crezi c te pasioneaz i apoi s iei o decizie informat pe care
domeniu vrei s te concentrezi. Dar, orice ai alege, nu lsa cteva
dezamgiri s te abat de la drumul pe care l-ai ales. n matematic, mai
ales, poi s munceti foarte mult i s i pice de multe ori ntr-un examen
probleme ciudate, sau pur i simplu s nu vezi soluia n ziua respectiv.
Faptul c nu eti premiat nu nseamn c pregatirea a fost inutil sau c
eti mai puin inteligent i ar trebui s renuni. Eti aceeai persoan ca
nainte de concurs i tot ceea ce poi face este s nvei din aceast
experien, poate s ii schimbi stilul de a nva, ori poate s ncerci din
nou mai mult i mai tare.
11.Ce crezi c ar trebui schimbat n nvmntul matematic
romnesc (acum c eti acolo i vezi altceva) ?
Nu prea mai tiu cum este nvmntul romnesc acum. tiu c
s-au fcut multe schimbri de cnd am termiant eu liceul, dar pot s spun
c sistemul n care am nvat eu a fost foarte bun i nu l-am apreciat
destul, pn cnd am cunoscut studeni din alte ri cu mai puin cultur
8
general i mai puin specializai n matematic. n general, sistemele
europene sunt foarte apreciate pe plan mondial, i nu prea neleg de ce se
ncearc o schimbare radical a nvmntului romnesc. ntr-adevar,
cred c este nevoie de o perspectiv mai practic n educaie, n care
teoria s fie nsoit de exemple concrete. Poate asta ar motiva mai mult
elevii s nvee i s aprecieze anii colii, pe cnd o schimbare total i nu
foarte bine organizat i va determina s trateze nvmntul n acelai
fel: neserios i neorganizat.
n numele tuturor cititorilor revistei noastre nu pot dect s
mulumesc din suflet domnioarei Anca Vcrescu pentru clipele pe care
i le-a rpit pentru a ni le drui , nu pot dect s sper c ali i ali mici
matematicieni i vor urma calea ascendent.
Prof.Lucian Dragomir,20 iulie 2006

Implementarea leciilor AEL n orele de matematic
Prof.Irina Avrmescu, Prof.Vasile Chi ,
coala cu clasele I-VIII nr.9, Reia
(Extrase din lucrarea cu acelai titlu prezentat la Simpozionul Didactica
Reia 2006)
Programul SEI pune la dispoziia beneficiarilor noi instrumente
didactice pentru utilizarea n coli, crescnd astfel calitatea procesului
educaional. Ofer un substitut pentru instrumentele sau experimentele de
laborator costisitoare sau periculoase pentru cei care le manevreaz.
AEL este coloana vertebral a programului SEI, oferind suport
pentru predare-nvare, evaluare i notare, administrarea, proiectarea i
monitorizarea coninutului. De asemenea, asigur mijloacele necesare
comunicrii i sincronizrii ntre centrele locale i regionale din cadrul
programului SEI.
AEL permite vizualizarea i administrarea unor tipuri vaste de
coninut educaional, precum: materiale interactive, tutoriale, exerciii,
simulri, jocuri educative. Biblioteca de materiale educaionale acioneaz
ca un gestionar de materiale: este adaptabil, configurabil, indexabil i
permite o cutare facil. Chiar i utilizatorii nceptori pot:
crea coninut (editor HTML ncorporat, editor de formule
matematice, editoare de teste i de dicionare);
importa/exporta coninut din fiiere, arhive de resurse utiliznd
standarde de mpachetare precum SCORM i IMS;
adapta sau edita coninut;
construi propriile cursuri din componente deja existente.
w
w
w
.
n
e
u
t
r
i
n
o
.
r
o
9
AEL este optimizat pentru nvare sincron, profesorul
controlnd n ntregime lecia, crend, coordonnd i monitoriznd
procesul educaional
AEL ofer de asmenea faciliti pentru nvarea asincron (n
ritmul fiecrui cursant), proiecte n colaborare i nvare la distan.
Testele sunt integrate cu fiele de studiu ale elevilor, sistemul
pstrnd evidena evoluiei fiecrui elev.
Prin AEL, profesorul poate s:
controleze transferul coninutului ctre elevi;
transmit individualizat momente de lecie ctre elevi, n funcie de
nivelul de capacitate sau cunotine ale acestora
administreze i monitorizeze testele;
urmreasc activitatea elevilor, monitoriznd ecranele de lucru i
rapoartele on-line;
n coala noastr suntem la nceputurile aplicrii leciilor AEL la
orele de matematic. Din scurta noastr experien am constatat c
elevilor le plac aceste lecii i folosesc cu uurin programul.
Exist lecii de geometrie interesante pe care le-am importat i
avem de gnd s le desfurm la leciile de recapitulare final. La clasa a
VI-a: Unghiuri, Triunghiul, Simetria fa de punct i dreapt,
Cercul, Poziii relative ale dreptei fa de cerc, Asemnare , la clasa
a VII-a, Construcia figurilor geometrice n spaiu, Construcia
corpurilor geometrice, Conul, Cilindrul, Trunchiul de con, etc.
De asemenea vrem s creem teste de verificare astfel nct unele
din lucrrile de evaluare s aib loc n laboratorul AEL.
n acest lucrare vom prezenta un exemplude proiect de lecie i
dou exemple de teste. Primul importat din fiierele existente i ultimele
create de noi.
Ne permitem doar s reamintim c pentru a pregti un material
didactic util i eficient trebuie s gsim un rspuns la urmtoarele
ntrebri:
Ce
Cu ce
Cui predm?
n ce condiii
Cu ce scop
Cum
10
n cazul instruirii programate rspundem astzi astfel la ntrebrile de mai
sus:
Ce? - Predm materia prevzut n curiculele colare
Cu ce? - Cu calculatoare PC corespunztoare.
Cui? - Celor ce doresc s nvee astfel.
n ce condiii? - Cu condiia n care societatea ne permite s folosim
calculatoare, deci coala este dotat cu programul AEL.
Cu ce scop? - Cu scopul de a obine rezultate ct mai bune n instruirea
i educarea elevilor.
Cum? - Prin programare didactic.


Proiect de lecie
Data: 4 aprilie 2006
Clasa a VII-a
Obiectul: Matematic Geometrie
Unitatea de nvare: Cercul
Tipul leciei: Dobndirea de noi cunotine
Obiective de referin:
1. S recunoasc i s defineasc elemente ale cercului: raz, arc, coard,
diametru
2. S recunoasc i s defineasc unghiul la centru i unghiul nscris n
cerc
3. S estimeze msurile unor unghiuri, distane, lungimi
4. S diferenieze informaiile dintr-un enun matematic dup natura lor
5. Dezvoltarea capacitii de a comunica utiliznd limbajul matematic
6. Dezvoltarea capacitii de explorare/investigare i rezolvare de
probleme
7. S fie ateni i s participe afectiv la lecie
8. S-i dezvolte interesul pentru studiul matematicii
Strategii didactice: conversaia, problematizarea, demonstraia
oral, explicaia
Mijloace de realizare: laborator AEL, cret colorat
Forme de organizare: frontal, individual
Desfurarea leciei
Se intr n sala AEL. Elevii i eu ne conectm la calculatoare.
Elevii au deja nsuite noiunile de cerc, raz, coard, diametru, unghi la
centru, unghi nscris n cerc.
w
w
w
.
n
e
u
t
r
i
n
o
.
r
o
11
Se explic c n prima parte a leciei vor vizualiza elementele
cercului, unghiul nscris i unghiul la centru. Timp estimat 15 minute.
n partea a doua vom vedea proprietile arcelor i a coardelor i
le vom scrie pe caiete, le vom demonstra oral i apoi elevii vor avea ca
tem pentru acas s le demonstreze pe caiet. Timp estimat 30 de minute.
Prima parte
Definim pe rnd cercul i elementele cercului cu proprietile lor.
Se notez pe tabl pe scurt. Se lanseaz lecia i prima component a
leciei. Elevii au pe monitor dou cuie i o sfoar pe care o leag de unul
din cuiele btute i traseaz cercul. Se vizualizez cercul i vor aprea pe
rnd elemntele cercului. Le vom crea, le vom citi definiiile pe ecran,
evalund corectitudinea definiiilor date de noi la nceputul leciei.
A doua parte
Se lanseaz componenta urmtoare a leciei. Vor aprea pe rnd
pe partea dreapt a ecranului figurile corespunztoare teoremelor pe care
le selectm. Notm pe caiete i pe tabl enunul teoremei. Analizm oral
modul de a o demonstra pe fiecare n parte. Vizualizm modificnd figura
c proprietatea nu are loc dac modificm o ipotez.
Evaluarea are loc prin aprecieri pozitive asupra contribuiei
fiecrui elev la desfurarea orei i aprecierile elevilor asupra modului de
concepere a leciei (cum au perceput-o, ce amanunte au retinut, etc.).
Observaie: I se permite pe rnd fiecrui elev s parcurg lecia
lucrnd efectiv pe monitor.
Aa cum s-a aratat i mai nainte, sistemul AELse preteaz nu
numai la predarea i fixarea noiunilor , dar i la verificarea msurii n
care ele au fostv nsuite de ctre elevi i la evaluarea acestora. n acest
sens sistemul AEL ofer asisten n vederea crerii de teste de verificare
i de evaluare.
Pentru a elabora un test , utilizatorul trebuie s parcurg trei pai:
crearea testului propriu-zis, crearea problemelor testului i crearea
variantelor de rspuns la aceste probleme.
Crearea testului propriu-zis necesit precizarea titlului testului, a
numelui autorului, a datei crerii descrierea testului, modul de parcurgere,
modul de revenire durata, etc.
Crearea problemelor testului necesita precizarea numelui
problemei , a gradului de dificultate (pe o scar de la 1 la 5), timpul
necesar, punctajul acordat i tipul problemei. n funcie de tipul problemei
se adaug tipul i numrul dorit de variante de rspuns.
12
Fr a intra n amnunte n legtur cu modul de creere a testelor
(acesta putnd fi gsit n Manualul de utilizare AEL) vom ilustra aceast
posibilitate de utilizare a sistemului AEL prin dou teste din materia de
gimnaziu.
Un prim test se adreseaz elevilor clasei a VI-a, la tema
Proprietile triunghiurilor. Se compune din 7 probleme, de tipuri
diferite i de diferite grade de dificultate.

TESTUL 1

PROBLEMA 1 (este de tipul adevrat/fals i voloreaz 1p)
Triunghiul cere are un unghi ascuit este ascuitunghic.
adevrat
fals.
PROBLEMA 2 (este de tipul adevrat/fals i valoreaz 1p)
Triunghiul care are dou unghiuri complementare este
dreptunghic.
adevrat
fals.
PROBLEMA 3 (este cu o singur variant corect i valoreaz 1p)
Un triunghi cu laturile de 5 cm, 7 cm i 10cm are semiperimetrul
de:
15 cm,
22 cm,
11 cm.
PROBLEMA 4( este cu o variant de rspuns i valoreaz 1p)
Fiind date triunghiurile echilaterale ABC i BCD, A D ,
msura unghiului ABD este de:

0
60 ,

0
120 ,

0
180 .
PROBLEMA 5 (este cu mai multe variante corecte , valoreaz 2p)
Dou dintre laturile unui triunghi isoscel sunt de 8 cm i respectiv
de 11 cm. Perimetrul triunghiului este de :
19 cm,
27 cm,
30 cm.
w
w
w
.
n
e
u
t
r
i
n
o
.
r
o
13
PROBLEMA 6 (este cu toate variantele corecte i valoreaz 2p)
Un unghi al unui triunghin isoscel are msura de
0
40 . Un alt
unghi al triunghiului poate avea msura de :

0
40 ,

0
70 ,

0
100 .
PROBLEMA 7 (este cu asociere de elemente i valoreaz 2p)
Un triunghi dreptunghic ABC are unghiul A drept i BC= 14 cm.
Asociai propoziiilor din prima coloan afirmaii din a doua coloan,
pentru a obine propoziii adevrate.
( )
0
30 m B = <
[ ] [ ] AB AC
( )
0
20 m C = < AC = 7 cm
( )
0
45 m B = < ( )
0
70 m B = < .
Al doilea test se adreseaz elevilor clasei a VII-a i le verific
cunotinele referitoare la tema Numere reale. Testul se compune din 9
probleme.
TESTUL NR. 2

PROBLEMA 1 (este de tipul adevrat/ fals i valoreaz 1p)
Numrul ( ) 0, 7 este raional.
adevrat,
fals.
PROBLEMA 2 (este de tipul adevrat/fals i valoreaz 1p)
Numrul
1
2
4
nu este iraional.
adevrat,
fals.
PROBLEMA 3 (este cu o singur variant corect i valoreaz 1p)
A treia zecimal exact a numrului 12 este:
2
4
6.

14
PROBLEMA 4 (este cu o singur variant corect i valoreaz 1p)
Media geometric a numerelor 3 i 27 este:
9
10
15.
PROBLEMA 5 (este cu o singur variant corect i valoreaz 1p)
Rezultatul calculului 45 80 180 + este:
9 5
- 5
5 .
PROBLEMA 6(este cu dou variante corecte i valoreaz 1p)
Valoarea expresiei ( ) 3 2 1 3
n
+ , unde nN, este:
2
3

( )
2 1 3 .
PROBLEMA 7 (este cu o singur variant corect , valoreaz 1p)
Raportul numerelor 3 24 i 12 6 este:
2
4

1
2
.
PROBLEMA 8(este cu o singur variant corect i valoreaz 1p)
Rezultatul calculului
( ) ( )
2 2
3 2 1 3 + este:
1
2 3
3 3 .





w
w
w
.
n
e
u
t
r
i
n
o
.
r
o
15
PROBLEMA 9 (este cu asociere de elemente i valoreaz 2p)
Asociai fiecrei fracii din prima coloan numrul real
corespunztor, din a doua coloan, obinut prin raionalizarea numitorului
acelei fracii.

6
3

6
3


4
24

2
2


( )
1
2

2 3 .
OBSERVAIE: n cazul fiecrui test, nota final se obine prin
nsumarea punctajelor problemelor rezolvate corect.

Exist evident plusuri i minusuri n aplicarea acestei metode de
predare.
Unul dintre elementele pozitive este faptul c este benefic
obinuirea elevilor cu mediul virtual care este din ce n ce mai frecvent n
era n care trim, era numit Era informaional.
Scopul unei lecii nu este numai nsuirea unor cunotine
cognitive ci i formarea unor deprinderi de asimilare a unor cunotine n
situaii mediale diferite prin dezvoltarea capacitilor de asimilare
creativ. Adaptarea la mediul n care trim i muncim nu presupune
numai nsuirea unor cunotine abstracte ci i familiarizarea cu
elementele de baz ale noii ere, cu cele mai noi tehnologii. coala trebuie
s fie mai mult dect o main de instruire.
Pe de alt parte unul dintre minusurile instruirii asistate pe
calculator este faptul c o or de rezolvri de exerciii i probleme
contribuie mai mult la dezvoltarea gndirii logice, a raionamentului
matematic. n perioada gimnaziului se formeaz anumite deprinderi de
calcul aritmetic i algebric, deci sunt necesare ct mai multe ore de
aplicaii.
Implementarea n programa de matematic a leciilor AEL trebuie
bine gndit, altfel exist riscul de a pierde ore preioase din instruirea
elevului. De exemplu noi am nceput unitatea de nvare Cercul i
elementele sale printr-o lecie n sala AEL. Lecia a mers bine dar
ulterior am constatat c bagajul de cunotine cu care a rmas elevul nu
era cel scontat. Pe de alt parte la clasa aVI-a, lecia fiind de fixare
rezultatele obinute au fost excelente: elevii au realizat figuri mult mai
16
corecte, au fixat foarte bine definiiile diferitelor categorii de triunghiuri,
a crescut rapiditatea cu care au recunoscut diferitele tipuri de triunghiuri.
Un alt punct n minus pentru leciile AEL este faptul c
elementele tridimensionale nu funcioneaz. Corpurile nu se rotesc, nu se
translateaz. Aceasta este ns o problem tehnic de funcionare a
programului care va fi remediat n timp.
Chiar cei care au elaborat programul au identificat cteva
probleme:
- teama de nlocuire a profesorului de ctre computer;
- teama de necunoscut;
- numrul insuficient de computere existent n coli, utilizarea lor
neadecvat, sau i mai ru neutilizarea lor.
Modaltile de contracarare a lor sunt:
- instruirea n utilizarea AEL este recunoscut n mod oficial ca
perfecionare didactic n cadrul programelor obligatorii de formare
continu a corpului profesoral;
-profesorii sunt stimulai material prin echivalarea unei ore
desfurate n laboratorul AEL cu 1,25 ore predate n sistem clasic.
La noi n coal n cadrul catedrei de matematic s-au luat msuri
ca n anul care urmeaz s realizm astfel planificarea nct s includem
leciile AEL n planificarea calendaristic i n planificarea unitilor de
nvare. Vom proceda la o analiz atent, n cadrul edinelor comisiei de
catedr, a momentului n care vom include lecia AEL n proiectare.
Avem de asemenea sarcina de lucru ca n timpul vacanei mari s
elaborm o serie de teste i lecii, s le introducem n computere cel trziu
n luna septembrie, pentru a deveni operaionale n anul colar urmtor.


Bibliografie:
[1] CNIV, Noi tehnologii de e-learning, Conferina Naional de
nvmnt virtual, Softwer educaional, Editura Universitii din
Bucureti, 2003 (ISBN 973-575-822-9)
[2] htpp://portal.edu.ro
[3] htpp://portal.edu.ro/adlic
[4] Frank, Helmar (1996): Klerigkibernetiko /
Bildungskybernetik. KoPd, Mnchen, 1996-1999. (Reeditat n
Meder/Schmid et al., Vol. II, 1999, 1 -239)
[5] Manual de utilizare AEL , versiunea 5.1, 2001-2005 SIVECO
Romnia SA
w
w
w
.
n
e
u
t
r
i
n
o
.
r
o
17
Matematica-azi
Opional la clasa a VI-a G

Prof. Humia Dorina i Mirulescu Maria,
Liceul Pedagogic C. D. LogaCaransebe

Durata cursului: 36 de ore anual
Argumente pentru alegerea opionalului:
Am ales acest opional pentru c urmresc:
- s lrgesc orizontul matematic al elevilor;
- s mresc palete de aplicaii pe care elevii le pot rezolva
utiliznd baza teoretic adunat pn acum;
- s ofer un alt punct de vedere asupra matematicii ncercnd s-o
prezint ntr-o form ct mai accesibil i plcut.
Obiective cadru urmrite n alctuirea programei
I. Dezvoltarea interesului i a motivaiei pentru studiul matematicii.

Obiective de referin
Elevul va fi capabil:
- s neleag importana studiului
pentru dezvoltarea raionamentului ;
- s perceap existena aplicabilitii
practice a matematicii i importana sa
n viaa de zi cu zi.
Activiti de nvare:
- analizarea unor probleme a cror
rezolvare are la baz observaii i
raionamente logice imediate;
- prezentarea unor probleme cu
coninut practic


II. Crearea climatului tiinific necesar dezvoltrii gndirii.

Obiective de referin
Elevul va fi capabil:
- s se mobilizeze pentru a putea
nelege i participa la activiti de
probleme propuse i de creare de
exerciii i probleme originale.
Activiti de nvare:
- analizarea rezolvrii unor probleme
date;
- propunerea de ctre elevi a unor
exerciii i probleme.


III. Cunoaterea i nelegerea conceptelor, terminologiei i
procedeelor de calcul specifice matematicii.

Obiective de referin
Elevul va fi capabil:
- s-i nsueasc noi concepte
matematice, terminologia aferent i
Activiti de nvare:
- analizarea unor probleme;
- notarea prescurtat a datelor (ce se
d, ce se cere);
18
procedeele de calcul specifice;
- s-i formeze obinuina de a recurge
la concepte i metode matematice n
abordarea unor situaii cotidiene sau
rezolvarea unor probleme practice.
- exerciii de rezolvare a problemelor
tip;
- redactarea rezolvrii unor probleme.


IV. Dezvoltarea capacitilor de explorare/investigare i rezolvare
de probleme.

Obiective de referin
Elevul va fi capabil:
- s aleag, din multitudinea de noiuni
nsuite i metode studiate, pe acelea
care l vor duce la rezolvarea unei
probleme anume dat.
Activiti de nvare:
- rezolvarea unor probleme ce implic
utilizarea succesiv a mai multor
metode i procedee studiate.


V. Dezvoltarea capacitii de a comunica utiliznd limbajul
matematic.

Obiective de referin
Elevul va fi capabil:
- s argumenteze rezolvrile fcute
utiliznd limbajul matematic adecvat;
- s colaboreze n cadrul unei echipe la
activiti specifice disciplinei.
Activiti de nvare:
- prezentarea n scris i oral a
rezolvrii unor probleme;
- elaborarea ntr-un grup de lucru a
rezolvrilor unor probleme dificile.


Coninuturi.
1. Calculul unor sume.
2. Probleme de numrare i combinatoric.
3. Probleme deosebite de divizibilitate.
4. Exemple i contraexemple n matematic.
5. Metoda reducerii la absurd n aritmetic.
6. Probleme de logic (distractiv).
7. Lucrri de verificare.
8. Recapitulare.
Modaliti de evaluare.
1. Teste de evaluare a cunotinelor.
2. Efectuarea unor lucrri individuale sau pe grupe cu autoevaluare.
3. Chestionarea oral pe tot parcursul anului.
4. Notarea n cadrul unor activiti practice prin observarea
activitii.
w
w
w
.
n
e
u
t
r
i
n
o
.
r
o
19
SEMESTRUL I

Nr.
crt.
Coninuturi Obiective
operaionale
Nr.
ore
Sptmna Obs.
I.
1.

2.


3.

4.
5.

Calculul unor
sume
Suma primelor n
numere naturale i
aplicaii ale ei.
Suma ptratelor
(cuburilor)
primelor n numere
naturale i
aplicaii ale ei.
Sume n care
intervin anumite
numere raionale.
Aplicaii.
Lucrare pentru
verificarea
cunotinelor.
Elevul va fi
capabil s:
- s recunoasc
tipul de problem
i metoda de
abordare a
rezolvrii;
- s generalizeze
metoda nvat
i la calcularea
altor sume.

2

2


2

1
1

(1) i (2)

(3) i (4)


(5) i (6)

(7)
(8)
Se
calculeaz
suma
primelor n
numere
pare,
(impare)
II.

6.
7.
8.
Probleme de
numrare i
combinatoric
Principiul cutiei.
Aplicaii.
Lucrare pentru
verificarea
cunotinelor.


- s recunoasc
problemele n
care se aplic
principiul cutiei;
- s poat rezolva
aceste probleme


2
2
1



(9) i (10)
(11) i (12)
(13)
Se insist
pe
compunere
de
probleme
care s se
ncadreze
n metoda
studiat.
III. Recapitulare - s poat
propune
probleme de
acest tip.
1 (14)
IV.

9.

10.
Probleme
deosebite de
divizibilitate.
Criteriul de
divizibilitate cu 7.
Criteriul de
divizibilitate cu
11.



- s poat utiliza
criteriile de
divizibilitate cu
7, 11, 13 n
rezolvarea unor
exerciii.


2
2


(15) i (16)
(17) i (18)

20

SEMESTRUL AL II-LEA

Nr.
crt.
Coninuturi Obiective
operaionale
Nr.
ore
Sptmna Obs.
11.

12.
13.
14.
15.

Criteriul de
divizibilitate cu
13.
Aplicaii.
Numere perfecte
Numere amiabile
Lucrare pentru
verificarea
cunotinelor
Elevul va fi
capabil s:
- s recunoasc
numerele
perfecte i
numerele
amiabile
1

1
1
1
1

(1)
(2)

(3)
(4)


(5)


Se calculeaz
suma primelor
n numere pare,
(impare)
IV.

16.


17.
18.
Exemple i
contraexemple n
matematic
Rolul exemplelor
i
contraexemplelor
n matematic
Aplicaii
Lucrare pentru
verificarea
cunotinelor


- s poat
construi
exemple i
contraexemple
pornind de la o
noiune dat.


2


1
1

(6)i (7)
(8)

(9)

Se insist pe
construirea de
exemple
i
contraexemple
pentru anumite
probleme date.
V.

19.
20.


21.
Utilizarea
metodei reducerii
la absurd
Descrierea
metodei
Aplicarea metodei
reducerii la absurd
n rezolvarea unor
probleme de
aritmetica
Lucrare pentru
verificarea
cunotinelor


- s recunoasc
problemele ce
se pot rezolva
folosind metoda
reducerii la
absurd;
- s poat
utiliza metoda
reducerii la
absurd.



1
2


1


(10)
(11) i (12)
(13)


w
w
w
.
n
e
u
t
r
i
n
o
.
r
o
21
VI.

22.
Probleme de
logic
(distractiv)
Rezolvarea unor
probleme de logic



- s poat
utiliza
raionamentul
logic n
rezolvarea unor
probleme.


2


(14) i(15)



VII. Recapitulare
final

3 (16), (17),
(18)


Standarde curriculare de performan

Standarde minimale.
1. S recunoasc suma primelor n numere naturale i s o poat
calcula pentru valori particulare ale lui n.
2. S recunoasc problemele care se pot rezolva utiliznd principiul
cutiei.
3. S contientizeze existena mai multor criterii de divizibilitate fa
de cele studiate la clas.
4. S perceap perfect ce nseamn exemplu i ce nseamn
contraexemplu n matematic.
5. S cunoasc etapele ce trebuie parcurse n aplicarea metodei
reducerii la absurd.
6. S perceap exact datele unei probleme de logic.
Standarde optime.
n plus fa de standardele minimale, elevul trebuie:
1. S poat calcula suma numerelor pare (impare) mai mici dect un
numr dat.
2. S rezolve probleme simple de numrare.
3. S poat aplica criteriile de divizibilitate cu 7, 11, 13 n
recunoaterea numerelor divizibile cu ele.
4. S poat construi singur exemple i contraexemple pentru
probleme date.
5. S recunoasc problemele ce pot fi rezolvate folosind metoda
reducerii la absurd.
6. S poat rezolva probleme de logic (distractiv) cu grad de
dificultate mediu.
Standarde de performan.
22
n plus fa de standarde optime elevul trebuie:
1. S poat calcula i alte sume n afar de cele relativ imediate.
2. S rezolve probleme folosind principiul cutiei.
3. S poat rezolva probleme n care intervin criteriile de divizibilitate
cu 7, 11, 13.
4. S utilizeze contraexemple n rezolvarea unor probleme.
5. S opereze concret cu metoda reducerii la absurd.
6. S rezolve probleme de logic (distractiv) cu grad mare de
dificultate.
Bibliografie.
1. Ion Ptracu, Constantin Preda, Complemente de matematic pentru
gimnaziu, Editura Cardinal, Craiova, 1994.
2. Liliana Niculescu , Teme de algebr pentru gimnaziu, Editura Cardinal,
Craiova, 1993.
3. Ioan Dncil, Matematica gimnaziului ntre profesor i elev, Editura
Corint, Bucureti, 1996.

Distane
Prof.Stniloiu Nicolae,Boca

Prezenta not fixeaz cteva idei de baz n calculul unor distane.
Pentru calculul distanelor de la un punct la un plan, urmtoarea
propoziie aduce n unele situaii o simplificare semnificativ a soluiei.
Propoziia 1: Dac este un plan, A un punct din plan, M un punct
exterior planului i E un punct pe segmentul [MA] a.. 0 > = k
EA
MA
,
atunci ) , ( ) , ( E kd M d = , unde prin d(M, ) se nelege distana de la
M la planul ..
Demonstraia acestei propoziii este foarte simpl i nu o vom mai
face ns voi sublinia importana aceste propoziii cu ajutorul unui
exemplu.
Aplicaia 1. Fie VABCD o piramid patrulater regulat avnd latura
bazei L=12 i nlimea VO=8. (vezi fig. 1). S se calculeze distana de la
A la faa (VBC).
n loc s construim perpendiculara din A pe planul (VBC) lucru care
ar complica figura, vom folosi propoziia anterioar, adic vom scrie c
( ) ( ) ( )) , ( 2 ) , ( ) , ( VBC O d VBC O d
OC
AC
VBC A d = = . Calculul distanei
w
w
w
.
n
e
u
t
r
i
n
o
.
r
o
23
( )) , ( VBC O d este foarte simplu aceasta fiind chiar nlimea triunghiului
dreptunghic VOE. n final vom obine c ( ) ( ) 6 , 9
10
8 6
2 , =

= VBC A d

Pentru calculul distanei dintre dou drepte neconcurente i neparalele
reamintim c distana dintre dou drepte neconcurente este lungimea
perpendicularei comune. Am observat c la foarte multe probleme de
acest tip se propun soluii bazate pe construcia perpendicularei comune
ceea ce complic uneori foarte mult soluia. n cele ce urmeaz propun o
metoda de calcul al lungimii perpendicularei comune bazat pe
construcia urmtoare: Se va construi mai nti un plan ce conine una din
cele dou drepte i este paralel cu cealalt dreapt (Acest plan este unic).
Lungimea perpendicularei comune este acum egal cu distana de la
aceast a doua dreapt la planul astfel construit. Pentru calculul acestei
distane se ia un punct pe aceast dreapt ales n mod convenabil aa nct
calculul acestei distane devine foarte simplu.
Aplicaia 2. Cu datele din problema anterioar, vom cere acum s se
calculeze: ( ) BC AV d ,
Pentru a da o soluie nu vom construi perpendiculara comun a celor
dou drepte ci vom observa c: ( ) VAD BC // i deci este suficient s
calculm ( ) ( ) VAD BC d , , distan care nu este altcineva dect nlimea
din E a triunghiului VEG i care este tot 9,6.
ncercai s folosii observaiile precedente n rezolvarea unor
probleme de acest tip i vei scpa de foarte multe necazuri.

A B
E
C
O
G
D
V
F
Fig 1
24
Funcii periodice
Prof.Mihai Monea
Colegiul Naional Decebal Deva

Acest articol nu conine neaprat lucruri originale , ci dorete s adune
cteva proprieti generale ale funciilor periodice. Motivul principal l
reprezint unele dintre subiectele propuse spre rezolvare la proba de
matematic a examenului de bacalaureat sau diferitele simulri din anii
2003-2005
Pentru nceput voi fixa urmtorul cadru general i vom vorbi despre
funcii periodice : f R R neconstante, cu perioada principal 0 T > .
Propoziia 1 Funcia f nu este strict monoton.
Demonstraie: Alegem
1 2
0 x x < < , astfel nct ( ) ( )
1 2
f x f x .
Conform axiomei lui Arhimede , exist
*
nN pentru care
3 1 2
x x nT x = + > . Evident ( ) ( )
1 3
f x f x = . Dac , de exemplu
( ) ( )
1 2
f x f x < atunci ( ) ( )
3 2
f x f x < i deci funcia nu este strict
monoton.
Propoziia 2 Nu exist ( ) lim
x
f x

.
Demonstraie: Fie x y , pentru care ( ) ( ) f x f y . Construim
irurile ,
n
x x nT = +
n
y y nT = + care au limita infinit. Avem
( ) ( ) ( )
n
f x f x nT f x = + = i ( ) ( ) ( )
n
f y f y nT f y = + = .
Evident ( ) ( ) ( ) ( ) lim , lim
n n
n n
f x f x f y f y

= = i cele dou limite sunt
distincte ceea ce este echivalent cu enunul propoziiei.
Propoziia 3 Dac f este continu n
0
x atunci f este continu i n
0
, x kT k + Z.
Demonstraie: Fie irul
n
x convergent la
0
x kT + . Fie
n n
y x kT =
care converge la
0
x . Atunci
( ) ( ) ( ) ( ) ( )
0 0 n n n
f x f x kT f y f x f x kT = = = +
Deci ( ) ( )
0 n
f x f x kT + ceea ce ncheie problema.
Propoziia 4 Dac f este continu pe [ ] 0,T atunci este continu pe
R.
w
w
w
.
n
e
u
t
r
i
n
o
.
r
o
25
Demonstraie: Continuitatea pe intervalul [ ] 0,T conduce la
continuitatea pe orice interval de forma ( ) , 1 , kT k T k +

Z conform
propoziiei anterioare, iar continuitatea pe R este consecin a relaiei
R ( ) , 1
k
kT k T

= +

Z

.
Avnd n vedere mrginirea funciilor continue pe intervale compacte,
putem spune c orice funcie periodic , continu pe [ ] 0,T este mrginit
pe acest interval i implicit pe R
Rezultatele urmtoare se adreseaz elevilor clasei a XII-a. Cadrul
general este urmtorul. Considerm funcia : f R R neconstant,
continu, cu perioada principal 0 T > . Construim funcia : F R R,
( ) ( )
0
x
F x f t dt =

.
Propoziia 5 Pentru orice xR avem ( )
x T
x
f t dt
+

= ( )
0
T
f t dt


Demonstraie: Considerm funcia : G R R, ( ) ( )
x T
x
G x f t dt
+
=

.
Derivnd avem ( ) ( ) ( ) 0 G x f x T f x = + = . Deci G este constant i
deci ( ) (0), G x G = x R ceea ce este echivalent cu relaia din enun.
Propoziia 6 Atunci
( )
( )
0
1
lim
T
x
F x
f t dt
x T
=

.
Demonstraie: Pentru orice x
+
R n N i [ ) 0,
x
r T astfel nct
.
x
x nT r = + . Atunci
2
0 ( 1)
( ) ( ) ( ) ... ( ) ( )
T T nT x
T n T nT
F x f t dt f t dt f t dt f t dt

= + + + +


0
( )
T
n f t dt = +


( )
x
nT r
nT
f t dt
+


0
( )
T
n f t dt = +


0
( )
x
r
f t dt

. Notm ( )
0
1
T
a f t dt
T
=

i atunci
( )
0
( )
x
r
F x naT f t dt = +

i
26
( )
0
( )
x
r
x
naT f t dt
F x
a a
x nT r
+
=
+

( )
0
x
r
x
x
f t dt ar
naT r

=
+

. Ultima expresie are


limita nul deoarece numrtorul este o cantitate mrginit iar numitorul
tinde la infinit cnd x
Propoziia 7 Funcia : G R R , ( ) ( ) G x F x ax = este periodic , cu
perioada principal T , unde ( )
0
1
T
a f t dt
T
=


Demonstraie: Avem
( ) ( ) ( ) ( ) ( ) G x T G x F x T a x T F x ax + = + + +

( ) ( ) ( ) ( )
0 0 0
0
x T x x T T
x
f t dt aT f t dt f t dt f t dt
+ +
= + = =

ceea ce trebuia
demonstrat.
Ca i consecin a aspectelor teoretice menionate mai sus propun
urmtoarea problem spre rezolvare:
Problem: Considerm funcia ( ) { } { } ( )
: , 2 1 f f x x x = + R R
unde { } x reprezint partea fracionar a numrului real x . Fie
( ) ( )
0
: ,
x
F F x f t dt =

R R . Atunci:
a) artai c ( ) ( ) 1 , f x f x x + = R
b) calculai ( )
0
lim
x
f x


c) demonstrai c f este continu pe R
d) calculai ( ) 1 F
e) determinai aria suprafeei cuprinse ntre graficul funciei, axa
Ox , dreapta 5 x = i 6 x =
f) calculai ( ) lim
x
f x


g) calculai ( ) lim
x
F x


h) demonstrai c funcia F nu are asimptote la
w
w
w
.
n
e
u
t
r
i
n
o
.
r
o
27
CANGURUL la Poiana Pinului
nv. Boulescu Florica,Reia


Organizat de Ministerul Educaiei i Cercetrii i Fundaia pentru
Integrare European Sigma, n perioada 27 iunie -3 iulie 2006 s-a
desfurat la Poiana Pinului, judeul Buzu, tabra naional Cangurul,
pentru elevii claselor II-VI, selecionai n urma probelor de baraj.
Am nsoit lotul judeului Cara-Severin, compus din trei elevi :
- Ciobanu Anca, coala cu clasele I-VIII Nr.2 Reia, clasa a II-a
- unea Adrian Marius, coala cu clasele I-VIII Nr.6 Reia,
clasa a IV-a
- Petrea Bogdan Andrei, coala cu clasele I-VIII Nr.8
Caransebe, clasa a V-a

n data de 29 iunie 2006, n tabr, a avut loc Concursul naional de
matematic Cangurul, la care au participat 236 de elevi din toate judeele
rii.
Reprezentanii judeului nostru au obinut rezultate absolut
remarcabile :

- Ciobanu Anca - premiul I (punctaj maxim), clasa a II-a
coala cu clasele I-VIII Nr.2 Reia

- Petrea Bogdan Andrei - premiul al II-lea, clasa a V-a
coala cu clasele I-VIII Nr.8 Caransebe


Programul taberei a cuprins diverse activiti instructiv-educativ-
recreative : ateliere de creaie, matematic distractiv, redactarea revistei
taberei, teatru interactiv, competiii sportive, jocuri, carnaval, la care
elevii au participat cu interes i plcere, avnd ocazia s se cunoasc mai
bine, s se mprieteneasc.
Felicitri tuturor elevilor participani i cadrelor didactice care i-au
pregtit i succes la ediiile viitoare ale Concursului de matematic
Cangurul.



28
Probleme rezolvate din RMCS nr. 16

Clasa a IV-a

IV.011 Ctul mpririi a dou numere este 7 , iar restul 13. Aflai cele
dou numere tiind c diferena lor este 613.
nv. Marioara Popescu , Reia
Rspuns: 713 i 100.

IV.012 n vacan , Marius a citit luni , mari i miercuri o carte ( a
terminat-o ! ) . Miercuri a citit cu 100 de pagini mai mult dect luni i
mari la un loc.Marius constat c miercuri a citit de trei ori mai mult i
nc 4 pagini dect n primele dou zile. Cte pagini a citit n fiecare zi i
cte pagini are cartea dac mari a citit de dou ori mai multe dect luni ?
nv. Marioara Popescu , Reia
Rspuns : 16,32,148,deci cartea are 196 de pagini.

IV.013 Daniel este ntrebat de tatl su : Ci elevi suntei n clas ? .
Daniel rspunde : Dac ar mai fi nc o dat pe ci elevi sunt i nc pe
jumtate i nc un sfert , mpreun cu nvtorul ar fi 100 de persoane!
Ci colegi are Daniel ?
Inst. Ozana Srin , Reia
Rspuns: 36 de colegi sunt n clas,deci Daniel are 35 de colegi.

IV.014 Cte caiete se pot cumpra n loc de un dicionar , tiind c un
dicionar cost ct 10 cri , 5 cri ct un penar i dou penare ct 5
stilouri , iar 10 stilouri ct 100 de caiete ?
nv. Ana Modoran , Reia
Rspuns: 50 de caiete.
IV.015 ntr-o clas sunt 30 de elevi. Ci biei i cte fete sunt n clas
tiind c dac ar fi cu doi biei mai puin , atunci jumtate din numrul
lor ar reprezenta de dou ori mai mult dect o treime din numrul fetelor ?
nv. Ana Modoran , Reia
Rspuns: 12 fete,18 biei.
IV.016 Mihai are 6 ani. Peste 6 ani , vrsta sa va fi exact de trei ori mai
mic dect a mamei sale, iar a bunicului de dou ori mai mare decat a
fiicei sale. Ce vrst are mama ? Dar bunicul ?
Petra-Ana Rogge , elev , Reia
Rspuns: Mama are 30 de ani , iar bunicul are 66 de ani.
w
w
w
.
n
e
u
t
r
i
n
o
.
r
o
29
IV.017 Dac mrim dou din laturile nealturate ale unui ptrat cu cte 6
cm, figura obinut va avea perimetrul de 44 cm. Aflai aria figurii
iniiale.
Inst. Lidia Todor , Caransebe
Rspuns:
2
64cm
IV.018 La un magazin de ceasuri, noul model are succes: n prima zi s-au
vndut 2/ 5 din total i nc 5 ceasuri. A doua zi s-au vndut 1/ 4 din rest
i nc 3 ceasuri. A treia zi s-au mai vndut 2/ 3 din cte rmseser i
nc 2 ceasuri. n raft mai sunt 4 ceasuri. Cte ceasuri au fost aduse spre
vnzare?
Inst. Lidia Todor , Caransebe
Soluie: Folosim metoda drumului invers: 4 2 6 + = reprezint
1
3
din
numrul de ceasuri rmase pentru a treia zi,aadar 6 3 18 = ceasuri
rmase pentru a treia zi. 18 3 21 + = ceasuri reprezint
3
4
din numrul de
ceasuri rmase pentru a doua zi. (21: 3) 4 28 = ceasuri au rmas deci
pentru a doua zi. 28 5 33 + = reprezint
3
5
din numrul iniial de ceasuri.
(33: 3) 5 55 = de ceasuri au fost aduse spre vnzare.
IV.019 Trei persoane au mers cu o main pentru care au pltit 350 000
lei; prima persoan a mers 15 km, a doua 25 km i a treia 30 km. Ct a
pltit fiecare pentru drumul parcurs?
nv. Mirela Ttar , Caransebe
Rspuns: 75000,125000,150000.
IV.020 Ioana propune prietenilor ei o problem :
Andrei este fratele meu ; ntr-o zi el mi spune : Am tot atia frai cte
surori am. . E adevrat , i rspund eu , dar eu am de dou ori mai
muli frai dect surori. . Ci biei i cte fete sunt n familia celor doi ?
* * *
Rspuns: 4 biei i 3 fete.

IV.021 Jerry se gsete la 20 de pai de adpostul su. Tom se gsete la
5 pai ( de pisic ! , eleva Irina Ciorogar a sesizat ! ) de oricel . Cnd
pisica face o sritur , Jerry face 3 pai , iar o sritur a pisicii este ct 10
pai de oricel . Poate Tom s-l prind pe Jerry ?
Concurs Bulgaria
30
Soluie: Pentru Tom sunt necesare 7 srituri pentru a ajunge la adpostul
oricelului;n 6 srituri de ale lui Tom,Jerry face 6 3 18 = pai,aadar
oricelului i rmn 2 pai pentru a ajunge la adpost ,iar cu al treilea pas
se va ascunde,deci Tom nu-l prinde.Suntei bucuroi? .
IV.022 Din Caransebe pornete spre Constana un automobil cu viteza
de 45 km/or. Dup trei ore, tot din Caransebe pornete spre Constana
un alt automobil care se deplaseaz cu 60 km/or. Aflai:
a)dup cte ore al doilea automobil l ajunge pe primul;
b)la ce distan de Caransebe se ntlnesc cele dou automobile.
nv. Mirela Ttar , Caransebe
Rspuns: dup 9 ore ; 540 km.
IV.023 Dublnd suma a dou numere naturale am obinut 150.Aflai
diferena numerelor tiind c primul este de patru ori mai mare dect al
doilea.
Inst. Adriana Ursu , Caransebe
Rspuns: 45.
IV.024 Un sac cu ciment cntrete de dou ori mai mult dect un sac cu
var. Dac 7 saci cu var i 7 saci cu ciment cntresc mpreun 525 kg , ct
cntrete un sac cu ciment ?
Inst.Mariana Ionescu , Caransebe
Rspuns: 50 kg.
Clasa a V-a

V.034. Fie 3 729 726 728 726 + + = a
S se stabileasc valoarea de adevr a propoziiei: a este ptrat perfect
i cub perfect .
Prof. Mariana Drghici , Reia
Soluie: a = 726 (1+728) +729 3; a = 729
2
i a = ( )
2
6
3 =( )
3
3
4
81 3 =
Propoziia este adevrat.


V.035 Care dintre numerele 124
228
i 626
171
este mai mare ?
Prof. Mariana Drghici , Reia
Soluie:
( ) ( )
228 171
228 228 3 684 4171 4 171 171
124 125 5 5 5 5 625 626

< = = = = = <
.
V.036 S se determine cifrele a, b, astfel nct numrul aabaa A = , scris
n baza 10 s fie divizibil cu 109.
Prof. Loreta Ciulu , Reia
w
w
w
.
n
e
u
t
r
i
n
o
.
r
o
31
Soluie: Numrul A poate fi scris sub forma:
( ) ( ) = + + = + = + = b a b a b aa aa A 9 109 2 109 101 100 11011 00 0
( ) b a b a b b a a 9 2 101 109 9 109 2 109 101 + + = + + = .
Rezult imediat c 2a-9b este divizibil cu 109.
Deoarece 9 0 ; 9 0 , , < < < < b a N b a , numrul 2a-9b este divizibil cu
109, dac 2a-9b=0. Rezult c a=9 i b=2, adic A=99299.
V.037 Se d numrul 2 5 4
3 2 1
=
+ + n n
A , n
a ) Gsii de cte ori apare cifra 9 n scrierea zecimal a lui A;
b ) Artai c numrul care reprezint suma cifrelor lui A este
divizibil cu 3 .
Prof. Zoran Ocanovici , Moldova-Nou
Soluie: a)

2 2
2 2 2 1
10 5 2 100...0 5 2 499...98
n
n n
A
+
+ +
= = = ,deci cifra 9 este
coninut de 2n+1 ori; b) se obine imediat c suma cifrelor lui A este
3(6 7) n + .
V.038 Se nir unul dup altul toate numerele naturale de la 1 pn la
2006 fr a lsa spaiu ntre ele i fr a pune virgul.Se obine numrul n
= 1234567891011...20052006.
a ) Determinai cte cifre are acest numr ;
b ) Aflai suma primelor 500 de cifre ale lui n .
Prof. Mariana Iancu , Oravia
Soluie : a ) Numrul cifrelor este 9 180 2700 4028 6917 + + + = ; b)
calcule destul de lungi conduc la suma cerut : 1907.
V.039 Determinai cte numere de 11 cifre ncep i se termin cu 2006.
Prof. Mariana Iancu , Oravia
Rspuns : 1000 de numere.
V.040 Gsii cel mai mare numr natural de forma abab care are cel mai
mic numr de divizori.
Prof. Vasile Huza , Coronini
Soluie: 9797.
V.041 Fie
2006 3 2
2 ... 2 2 2 + + + + = N . tiind c x este suma dintre
restul mpririi lui N 2006 la 10 i un numr prim , iar k este un numr
natural par astfel nct x k = 2006 ,determinai valorile lui x .
Prof. Georgeta Bihoi , Reia
32
Soluie: Avem :
2007
2 2 2 N N N = = ;deducem imediat :
( ) 6 u N = ,de unde ( 2006) 0 u N = .Deducem apoi c x este numr
prim ( ! ) i n final : { } 17, 59 x .
V.042 a ) Este numrul 2007 2006 ... 3 2 1 + = a ptrat perfect ?
b ) Este numrul
2006 2001 ) 2000 ... 3 2 1 (
2000
+ + + + + = b cub perfect ?
Prof. George Pascariu , Bozovici
Soluie: a) ( ) 0 7 7 u a = + = ,deci a nu este ptrat perfect; b) Observm c
x =
2000
(1 2 3 ... 2000) 2001 + + + +
667 3
(10 2001 ) = i urmtorul cub
perfect este
( )
3
667
10 2002 y = ( !! ) , deducem c b nu este cub
perfect.
V.043 Artai c n = 1 + 3 + 5 + ... + 2005 este ptrat perfect.
Prof. Nicolae Dragomir , Tudor Deaconu , Reia
Soluie: De exemplu,putem scrie: 2005 2003 ... 5 3 1 n = + + + + + ,de
unde 2 (1 2005) (3 2003) ... 1003 2006 n = + + + + = ,deci
2
1003 n = .(Generalizai rezultatul ! ) .
V.044 O foaie de hrtie este rupt n 3 buci ; una dintre acestea se rupe
deasemenea n 3 buci , apoi o bucat din cele avute se rupe din nou n 3
buci . Continund procedeul,se poate obine un total de 2006 de buci ?
Dac da , dup cte operaii ? Dar 2007 de buci se pot obine ?
Prof. Heidi Feil , Oelu-Rou
Soluie: Dup n operaii avem 2( 1) 3 n + buci.Din
2( 1) 3 n + 2006 = avem o egalitate ntre un numr par i unul
impar,adic absurd.Din 2( 1) 3 2007 n + = obinem ns 1004 n = .
V.045 Se consider numrul abbc A = .
a ) Cifra a se nlocuiete cu cifra d i se obine numrul dbbe B = ,
astfel nct d c e = .Determinai cte numere A exist pentru care
2006 = B A ;
b ) Schimbnd ordinea cifrelor lui A obinem un numr C. Artai c
2006 C A .
Prof.Lucian Dragomir ,Oelu-Rou
w
w
w
.
n
e
u
t
r
i
n
o
.
r
o
33
Soluie: 1000 ( ) ( ) 2006 2, 6 a d c e a d c e + = = = ;deducem
imediat c { } 6, 8, ( , ) (6, 0), (7,1), (8, 2), (9, 3) d a c e = = .Pe de alt
parte { } 0,1, 2,..., 9 4 9 36 b = de numere posibile.
V.046 Gsii ptratele perfecte de forma aabb .
Prof.Romana i Ioan Ghi , Blaj
Soluie: aabb
2
, 3 xy x = ;cum aabb este multiplu de 11,deducem c i
xy are aceeai proprietate,de unde x y = .Se ajunge acum uor la
2
7744 88 . =
V.047 Dac a < b sunt numere naturale consecutive , artai c :
b ab ab ab ab b
n n n
=
2 2 1
... , n , n 2.
Prof. Afilon Moica , Zvoi
Soluie: Conform ipotezei avem 1 b a = ;n membrul stng avem
succesiv:
1 1 1 1 2 2 2
( ) , ( ) ,...
n n n n n n n n
b ab b b a b b ab b b a b

= = = = ,
2
( ) . b ab b b a b = =
V.048 Determinai numerele naturale nenule care mprite la 5 dau ctul
a i restul b i mprite la 9 dau ctul b i restul a . Cte soluii are
problema ?
Prof. Adriana Dragomir , Oelu-Rou
Soluie: 5 , 5 n a b b = + < i 9 , 9 n b a a = + < ;deducem acum: 2 a b = i
considerm { } 1, 2, 3, 4 b .Obinem numerele : 11,22,33,44.
Clasa a VI-a
VI.034 S se determine trei numere prime avnd suma egal cu 44 ,
tiind c unul dintre ele este de forma aa .
Prof. Emilia-Dana Schiha , Berzasca
Soluie: 2 , 11 , 31. (singurul numr prim de forma aa este 11 )
VI.035 S se calculeze suma tuturor numerelor de forma ababab tiind
c fiecare astfel de numr are exact 24 de divizori i ab este numr
prim.
Prof. Marius andru , Reia
Soluie : Folosim : 10101 3 7 13 37 ababab ab ab = = ; ajungem la
numerele : 131313 i 373737,iar suma cerut 505050.
34
VI.036 Bisectoarele a dou unghiuri adiacente formeaz un unghi cu
msura de 60
0
.Aflai msura fiecrui unghi tiind c msura unuia este de
patru ori mai mare dect a celuilalt.
Prof. Vasile Huza , Coronini
Soluie :
0 0
24 , 96
VI.037 Gsii numerele naturale x i y , x y , care satisfac :
738 3 3 = +
y x

Prof. Vasile Huza , Coronini
Soluie:
2
3 (1 3 ) 3 82
x y x
+ = ;deducem 2, 6 x y = = (artai cum ! ).
VI.038 n triunghiul ABC ) ( ) ( C m B m + reprezint 0,8 din
) ( A m ,iar
0
10 ) ( ) ( = C m B m .Aflai:
a ) Msurile unghiurilor tringhiului ABC;
b ) Msura unghiului format de bisectoarea lui BAC i nlimea din B.
Prof. Ion Belci , Reia
Soluie : a)
0 0 0
100 , 45 , 35 ; b) considerm
, , .( BH AC H AC iar AD bisectoarea lui BAC X .Notm
{ }
0 0 0
( ) 90 , ( ) 50 ( ) 40 AD BH E m EHA m DAC m AHE = = = = X X X
.
VI.039 Dup o mrire de 2 %, apoi alta de 3 % , urmat de o reducere a
preului cu 4 % , o imprimant cost 504,288 lei ( noi ). Ct a costat
iniial imprimanta ?
Prof. Emilia Dana Schiha , Berzasca
Soluie : 500 lei.
VI.040 Cte triunghiuri cu laturile de lungimi numere naturale exist
tiind c numai o latur a unui astfel de triunghi are lungimea strict mai
mare dect 3 ?
Prof. Romana i Ioan Ghi , Blaj
Soluie: 3, , , {1, 2, 3} c c a b > N .Deosebim cazurile:
(1) 1 a b = = (absurd deoarece nu e satisfcut condiia 1 1 c + > );
(2) 2 a b = = (absurd din motive analoage);
(3) 3 {4, 5} a b c = = ;avem deci dou triunghiuri ( isoscele);
(4) 1, 2 a b = = (absurd) ; (5) 1, 3 a b = = (absurd);
(6) 2, 3 4 a b c = = = .Exist deci trei triunghiuri care satisfac enunul.

w
w
w
.
n
e
u
t
r
i
n
o
.
r
o
35
VI.041 17 puncte situate n plan determin 46 de drepte. Aflai cte dintre
punctele date sunt coliniare.
Prof. Liviu Smarandache , Craiova
Soluie: Dac n puncte din cele date sunt necoliniare,atunci 17- n sunt
coliniare.Numrul dreptelor determinate de cele 17 n puncte coliniare
este 1.Cele n necoliniare determin
( 1)
2
n n
drepte i mai avem
(17 ) n n drepte care se obin unind un punct din cele necoliniare cu cte
unul din cele coliniare.Din
( 1)
1 (17 ) 46
2
n n
n n

+ + = deducem
3 n = ,deci 14 puncte sunt coliniare.
VI.042 Artai c numrul 06 ... 200 = A nu poate fi ptrat perfect
( indiferent cte cifre de 0 avem ) .
Prof. Adriana Dragomir , Oelu-Rou
Soluie : Se tie c suma cifrelor unui numr natural este congruent
modulo 9 cu numrul (reinei dac nu tiai ! ) ;se arat astfel c un
numr natural n nu este ptrat perfect dac 2, 3, 5, 6, 8(mod9) n .n
cazul nostru 2006 8(mod9) . ( Sau,fiind numr par,A ar fi ptrat
perfect dac n primul rnd s-ar divide cu 4).
VI.043 n ptratul unui numr a , de mai multe cifre , pe locul zecilor se
afl cifra 7. Ce cifr se afl pe locul unitilor n numrul a
2
?
Concurs Ungaria
Soluie: Artm c ultima cifr a ptratului unui numr ntreg este 6
atunci cnd pe locul zecilor este o cifr impar: fie c ultima cifr a
numrului a ;avem atunci c a + c este par , iar a c se divide cu 10,de
unde
2 2
( )( ) a c a c a c = + se divide cu 20,deci ultima cifr a lui
2
a i
2
c este aceeai,iar cifra aflat pe locul zecilor este sau par n ambele
cazuri sau impar n ambele cazuri.
n ptratul numerelor formate dintr-o singur cifr,pe locul
zecilor se afl o cifr impar numai n cazul
2
4 16 = i
2
6 36 = ,iar ultima
cifr este 6.De aici se deduce c
2
a are pe locul zecilor o cifr impar
numai dac se termin cu 4 sau cu 6 i n toate aceste cazuri ultima cifr a
lui
2
a este 6. S mai remarcm c exist numere care satisfac enunul,de
exemplu
2
24 576. =
VI.044 n anul 1066 la Hastings s-a dat o mare btlie ntre otile saxone
i cele normande. Pe rm , saxonii au format un ptrat, iar pe plaj
36
normanzii au format alt ptrat . Se spune c normanzii ntreceau pe saxoni
cu 500 de pedestrai i 12 clrei. Atacnd cu mult curaj , saxonii au ucis
jumtate din cotropitori , ei nepierznd dect o cincime din ostai.La
sfritul btliei cele dou tabere aveau exact acelai numr de oameni.
Ci saxoni i ci normanzi au luat parte la lupt ?
* * *
Soluie : Notm cu x numrul normanzilor de pe o latur a ptratului lor i
cu y cel al saxonilor de pe o latur a formaiei saxone de lupt ;conform
ipotezei avem :
2 2
512 x y = sau ( )( ) 512 x y x y + = . Descompunem
pe 512 n factori primi i analizm mai atent datele problemei(jumtate
din x este apropiat de
2
y !! ) ; studiem cazurile ce apar i ajungem la
unica soluie : 36, 28 x y = = .La nceputul btliei normanzii erau
1296,iar saxonii 784.
VI.045 Pe un cerc se aeaz la ntmplare 5 numere naturale a cror sum
este egal cu 18. Artai c exist cel puin dou numere alturate a i b
astfel nct a + b 8.
* * *
Soluie : Dac 7, 7, 7, 7, 7 a b b c c d d e e a + + + + + ,atunci :
2( ) 35 a b c d e + + + + ,contradicie cu 18 a b c d e + + + + = .
VI.046 Considerm segmentul [AB] de lungime 1 m . n interiorul
segmentului desenm cu culoare roie punctele C
1
, C
2
, , C
24
, care
mpart [AB] n segmente congruente de lungime 4 cm. Colorm apoi cu
albastru punctele D
1
, D
2
, , D
19
care mpart [AB] n segmente
congruente de lungime 5 cm. Precizai cte puncte albastre se suprapun
peste puncte roii i cte segmente de lungime 1 cm apar pe desen.(enun
corectat).
Prof. Mihai Monea , Deva
Soluie : Un punct rou i unul albastru se suprapun dac i numai dac
exist { } { } 1, 2,..., 24 , 1, 2,...,19 k l astfel nct
k l
AC AD = .Cum
4 , 5 4 5
k l
AC k AD l k l = = = ,deci k e multiplu de 5 i l e multiplu de
4 .Exist 4 puncte care se suprapun :
5 4 10 8 15 12 20 16
, , , C D C D C D C D = = = = . Pe segmentul [ ]
5
AC avem
dou segmente de lungime 1cm :
[ ] [ ]
1 1 3 4
, C D D C ;situaia se repet de 5
ori,avem deci 10 segmente de lungime 1cm.
w
w
w
.
n
e
u
t
r
i
n
o
.
r
o
37
VI.047 Determinai toate fraciile de forma
b
a
( a , b * ) cu
proprietatea c , adunnd i la numitor i la numrtor acelai numr
x > 0 , obinem o putere cu exponent natural a fraciei iniiale .
Cutm fraciile de forma
a
b
(
*
, a bN ) cu proprietatea c exist nN
astfel nct (1) .
n
a x a
b x b
+
=

+


Singurele fracii care verific (1) sunt cele echiunitare. ntr-adevr, putem
scrie
( ) ( ) 0
n
n n n n n n
a x a
a x b b x a ab a b xb xa
b x b
+
= + = + + =

+


( ) ( )
1 1
0
n n n n
ab b a x b a

+ =
i deducem c singura posibilitate de realizare a egalittii este ca
numerele a i b s fie egale.n caz contrar, dac , a b < atunci
( ) ( )
1 1
0,
n n n n
ab b a x b a

+ >

iar dac , a b > atunci
( ) ( )
1 1
0.
n n n n
ab b a x b a

+ <
n concluzie, fraciile
a
b
(
*
, a bN ) care verific relaia (1) sunt cele
pentru care a b = i numai acestea.
VI.048 Fiecare element al mulimii A = { 1 , 2 , 3 , , 100 } se
coloreaz cu una dintre culorile rou , galben sau albastru , respectnd
urmtoarele reguli;
a ) suma dintre orice numr galben i orice numr albastru este
divizibil cu 3 ;
b ) suma oricror dou numere roii este divizibil cu 3.
1 ) Artai c numrul 3 este rou ;
2) Calculai suma tuturor numerelor care nu sunt roii .
Prof.Marius Damian , Brila
Soluie: a) Dac, prin absurd, numrul 3 nu ar fi rou, atunci el ar fi
galben sau albastru.
Dac 3 ar fi galben, atunci, din condiia i) deducem c numerele 6, 9,
12, ..., 99 sunt galbene sau albastre, iar celelalte numere: 1, 2, 4, 5, 7, 8,
..., 97, 98 sunt roii. Cum 1 4 5 + = care nu este multiplu de 3, se
contrazice condiia ii).
38
Dac 3 ar fi albastru, printr-un raionament asemntor, obinem din
nou contradicie.
n concluzie, numrul 3 este rou
b) Din a) rezult c toate numerele: 3, 6, 9, ..., 93, 96, 99 sunt roii. In
plus, acestea sunt singurele numere roii, deoarece, n caz contrar, dac un
numr nedivizibil cu 3 ar fi rou, atunci s-ar contrazice condiia ii).
Suma numerelor care nu sunt roii este aadar:

( ) ( ) ( )
( )
1 2 ... 100 3 6 ... 99 1 2 ... 100
100 101 33 34
3 1 2 ... 33 3 3367.
2 2
S = + + + + + + = + + +

+ + + = =

Clasa a VII-a
VII.034 n triunghiul ABC bisectoarele interioare ale unghiurilor B i C
intersecteaz mediana [ ] AD n M i N astfel nct
[ ] [ ] [ ] ND MN AM . S se determine aria triunghiului
ABC tiind c NC=14 cm.
Prof. Mariana Drghici , Reia
Soluie: Din
3
2
=
AD
AN
i [ ] AD -median n triunghiul ABC, rezult c N
este centrul de greutate al triunghiului ABC.
Fie [CE bisectoarea unghiului ACB i EAB. n triunghiul ABC, [CE
este bisectoare i median, N CE , deci este isoscel cu [ ] [ ] BC AC .
E fiind mijlocul laturii [ ] AB , rezult CE= 21
2
3
= CN cm.
n triunghiul ABD aplicnd teorema bisectoarei obinem
2
1
= =
MD
AM
BD
AB
, de unde AB=
1
4
BC .
Aplicnd teorema lui Pitagora n triunghiul ACE obinem AB = 2 7 .
S
7 21
2
7 2 21
2
=

=
CE AB
ABC
.
VII. 035 Pentru cte valori naturale ale lui n, expresia

n
n

+
7 2
2 7
este numr ntreg?
Prof. Loreta Ciulu , Reia
w
w
w
.
n
e
u
t
r
i
n
o
.
r
o
39
Soluie: Fracia dat se poate scrie
7 2
7 5
2
7 2
7 2

n n
n
, care este
numr ntreg dac 7 2 n divide pe 7 5 . Rezult c 7 2 n
poate fi 7 , 7 + , 7 5 i 7 5 + , adic
7 7 7 7 2 = = = n n n .Analizm i celelalte cazuri posibile
i ajungem la : { } 343 ; 63 ; 7 n adic pentru trei valori naturale.
VII. 036 Se consider un triunghi ABC i punctele M , N , P pe laturile
[BC] , [AC] , respectiv [AB] astfel nct :
3
1
= = =
AB
AP
CA
CN
BC
BM

Dac E este mijlocul lui[NP] i F mijlocul lui [BC] , demonstrai c EF
este paralel cu AM i AM EF
2
1
= .
Prof. Nicolae Stniloiu , Boca
Soluie: Fie T un punct pe (BC) astfel ca
1
3
CT CB = .Deducem c TNAP
este paralelogram,deci AT trece prin mijlocul lui(PN),adic A,E,T sunt
coliniare i E este mijlocul lui (AT);cum F este mijlocul lui(MT) , avem
c EF este linie mijlocie n ATM , de unde concluzia e imediat.

VII. 037 Rezolvai n ecuaia : 0 17 5 3 = + x y xy .
Prof. Emilia-Dana Schiha , Berzasca
Soluie simpl : Scriei ecuaia sub forma ( 3)( 5) 2 x y + = i nu uitai
c suntei n Z(dac cumva nu ai fost ateni ) .
VII. 038 Demonstrai c dac a , b , c sunt lungimile laturilor unui
triunghi , atunci : 1 < + +
c
a
b
c
a
b
a
c
c
b
b
a
.
Prof. Nicolae Dragomir,Tudor Deaconu,Reia
Soluie: Calcule multe , dar rezultatul merit.Aducem la acelai
numitor,adugm i scdem la numrtor produsul abc i numrtorul va
deveni ( )( )( ) a b b c c a .Folosim acum inegaliti ntre laturile unui
triunghi.
VII.039 Cte numere de forma abcde sunt divizibile cu 11 i satisfac
a e b d + = + . ?
Prof. Nicolae Stniloiu , Boca
40
Soluie: Cam mult de numrat,dar hai s vedem.Fie
N a b c d e = + + ; trebuie s remarcm rapid c abcde se divide cu
11 dac i numai dac N se divide cu 11 ( de ce ? facei N - abcde ).S
mai sesizm c N c = ,deci exist attea numere din cele cutate cte
numere abed satisfac a e b d + = + i sunt divizibile cu 11, 0 a .Vom
obine cu ceva calcule 819 204 615 = numere.
VII. 040 Determinai numerele reale x
3
2
pentru care :

3
2
9
2
3
=
x
x .
Prof. Ion Belci , Reia
Idee: Ecuaia se poate scrie
2 2
3 3 9
x
x = ;se mparte cu 3 i se noteaz
2
3 9
x
y = ,de unde { } 0,1 y ;continuai.
VII. 041 Determinai x
*
i y prim pentru care 3
1
2
= +
x
y
x
.
Prof. Dumitru Btineu-Giurgiu , Bucureti
Soluie: Ecuaia se poate scrie : (3 ) y x x = .Cum y este prim ,deducem:
1, 2 x y = = sau 2 x y = = ( fals) .
VII.042 Determinai x care satisface :
x
2 2 ... 2 2 1
499 1 0
= + + + + .
Prof. Lenua Andrei , Craiova
Soluie:
500
2 2 250.
x
x = =
VII.043 Artai c ecuaia 2007
2 2 2
= + + z y x nu are soluii n .
Prof. Adriana Dragomir , Oelu-Rou
Soluie: Pentru orice
2
, (mod8) x x k Z ,unde { } 0,1, 4 k .Deducem
c
2 2 2
( ) (mod8), {0,1, 2, 3, 4, 5, 6} x y z y y + + .Pe de alt parte avem
2007 7(mod8) .
VII.044 Determinai restul mpririi la 7 a numrului

2006 2007
78 76 + = A .
Prof. Heidi Feil , Oelu-Rou
w
w
w
.
n
e
u
t
r
i
n
o
.
r
o
41
Soluie: O posibil rezolvare este urmtoarea:
2007
(77 1) 1(mod7) ,iar
2006
(77 1) 1(mod7) + ,aadar restul cerut
este 0.
VII.045 O mulime A avnd 4 elemente numere raionale se numete
interesant dac : x A 2x A sau
2
x
A . Notm
S
n
= { 1 , 2 , 3 , ... , n } , n
*
.
a ) Determinai cte submulimi interesante are S
10
;
b ) Determinai cel mai mic numr natural n pentru care S
2n
are
cel puin 2006 de submulimi interesante.
Prof.Lucian Dragomir , Oelu-Rou
Remarc:La aceast problem am primit o singur ncercare de
soluie,astfel c ateptm nc(poate chiar comentarii,poate chiar ale
profesorilor).
VII.046 Fie ABC un triunghi , I centrul cercului su nscris i
} {D BC AI = . O dreapt perpendicular pe AI intersecteaz (AB) i
(AC) n punctele P i respectiv Q;notm cu M i N simetricele lui P i Q
fa de dreptele BI i respectiv CI.
Demonstrai c MD = DN dac i numai dac AB = AC.
Prof.Marius Damian , Brila
Soluie: Dreapta AI este mediatoarea segmentului PQ; atunci:
(1) . IP IQ =
innd cont c ( BI
i (CI sunt bisectoare,
deducem c punctele M i N
sunt situate pe latura BC.

Totodat, BI este mediatoarea
segmentului ( ) PM ,
de unde rezult:
(2) , IP IM = iar CI este
mediatoarea segmentului
( ) QN , deci: (3) . IQ IN =
Din (1), (2) i (3), obinem , IM IN = deci triunghiul IMN este isoscel.
Trecem la justificarea celor dou implicaii.
42
" ": Dac , MD DN = atunci, innd cont c IMN . este isoscel,
rezult c , ID MN adic . AD BC De aici i din faptul c ( AD
este bisectoarea unghiului BAC, urmeaz c triunghiul ABC aste isoscel,
cu . AB AC =
" ": Dac , AB AC = atunci, innd cont c ( AD este bisectoarea
unghiului BAC, rezult c , AD BC adic . ID MN De aici i din
faptul c IMN . este isoscel, urmeaz c . MD DN =
VII.047 . a ) Artai c dac a, b , c i 6 = + + c b a ,
atunci 12
2 2 2
+ + c b a
b ) Rezolvai ecuaia : 6 5 2 4 2 3 3 = + + + + + y x x y y x
Prof. Petrior Neagoe , Anina
Soluie : a ) Exist mai multe posibiliti de a obine inegalitatea dat ; de
exemplu , cu inegalitatea CBS :
( ) ( )
2 2 2 2
3 36 1 1 1 c b a c b a + + = + + ; concluzia e imediat .
Evident , egalitatea se obine dac i numai dac a = b = c .
b ) notm radicalii din membrul stng cu a , b , respectiv c ; deoarece
6 = + + c b a , deducem 12
2 2 2
+ + c b a ; efectund ns calculele
avem c 12
2 2 2
= + + c b a , aadar egalitate , care se obine pentru a =
b = c = 2 . Avem imediat : x = 3 i y = 2 .
VII.048 Demonstrai c orice triunghi cu aria mai mare dect 1 nu
poate avea perimetrul mai mic dect 4 .
Prof.Lucian Dragomir , Oelu-Rou
Soluie : cu notaiile uzuale avem p , p- a , p- b , p- c > 0 , deci
putem folosi inegalitatea mediilor :
2 2
) (
c b a p p
a p p
+
=
+
i
2
) )( (
a
c p b p ; folosind formula lui Heron avem astfel c aria
triunghiului satisface :
4
) (
) )( )( (
c b a
c p b p a p p S
+
< =
; la fel
( alegnd altfel ) obinem
4
) ( a c b
S
+
<
i
4
) ( b a c
S
+
<
Deducem :
2
3 3
ca bc ab
S
+ +
< < , de unde 6 > + + ca bc ab
w
w
w
.
n
e
u
t
r
i
n
o
.
r
o
43
Si folosim( ) ) ( 3 2 2 2
2 2 2 2
ca bc ab ca bc ab c b a c b a + + + + + + + = + +
de unde 18 ) (
2
> + + c b a a + b + c > 18 > 4 .
Clasa a VIII-a
VIII.034 S se determine numerele naturale ptrate perfecte de forma
4n
2
-15, nN.
Prof. Mariana Drghici , Reia
Soluie: Fie m
2
15 4
2
= n 15 = (2n-m) (2n+m). Cum nN,
avem cazurile:

= +
=
5 2
3 2
m n
m n
sau

= +
=
3 2
5 2
m n
m n
sau

= +
=
1 2
15 2
m n
m n
sau

= +
=
15 2
1 2
m n
m n
,
de unde n= 2, m= 1 sau n = 4 , m = 7 , deci m
2
= 1 sau m
2
= 49 sunt
numerele naturale cerute n enun.
VIII.035 Fie numerele reale x, y cu proprietile: 0 2 6 = + y x i
[ ] 4 ; 2 x .S se determine numrul
17 2 8 4 4
2 2 2 2
+ + + + + + = y x y x x y x a .
Prof. Loreta Ciulu , Reia
Soluie: a ( ) ( ) ( )
2 2 2 2
1 4 2 + + + + = y x y x
Din 0 2 6 = + y x gsim y x 6 2 = + i 6 6 4 = y x .
Cum 4 2 x , rezult 6 2 0 + x i 6 6 0 y sau 1 0 y .
Deci, ( ) ( ) [ ] ( ) ( ) = + = + + + =
2 2 2 2 2 2
1 37 37 1 1 6 6 y y y y y y a
( ) ( ) 37 37 1 37 1 37 37 1 37 = + = + = + = y y y y y y
VIII.036 Determinai numerele reale
n
x x x ,..., ,
2 1
care satisfac:

= + + +
+
+
+
+

na x x x
x x x
x x x
x x x
x x x
n
n
n n
...
0 2 5 3
0 2 5 3
......... .......... ..........
0 2 5 3
0 2 5 3
2 1
2 1
2 1
4 3 2
3 2 1
, a .
Prof. Nicolae Stniloiu , Boca

44
Soluie: Adunm inegalitile date i avem 0 0 ,aadar nici o inegalitate
nu poate fi strict;deducem c toate sunt egaliti i apoi avem :
1 2 2 3
2 3 2 3
1 1 2
3( ) 2( )
3( ) 2( )
..................................
3( ) 2( )
n
x x x x
x x x x
x x x x
=


Dac oricare dou numere
k
x sunt diferite,prin nmulirea egalitilor
anterioare ajungem la o egalitate absurd;deducem n final
c: , 1,
k
x a k n = = .



VIII.037 Determinai cte perechi ( x , y ) de numere ntregi satisfac:
y x y x = + 3
2 2

Prof. Adriana i Lucian Dragomir , Oelu-Rou
Soluie: nmulind egalitatea dat cu 4 avem:
( ) ( )
2 2
2 3 2 1 10 x y + + = .( n Zavem doar 1 9 10, 9 1 10 + = + = )
Ajungem n final la 8 soluii.
VIII.038 Fie ABCD un dreptunghi .Artai c nu exist un punct M n
spaiu , nesituat n planul dreptunghiului dat ,astfel nct :
) ( ABCD MD ,
0
60 ) ( = MBC m i
0
30 ) ( = MBA m .
Prof. Ion Belci , Reia
Idee:Evident,reducere la absurd i apoi e cam simplu.
VIII.039 Determinai funciile f : care satisfac :
f ( x 2 ) 7 2x i f ( 2 x ) 2x 1 , x .
Prof. Mirela Rdoi , Reia
Soluie: Notai 2 ( ) 2 3 x t f t t = + ,apoi 2 x t = ;se ajunge la :
( ) 2 3 f t t = + .
VIII. 040 Fie ABC un triunghi dreptunghic n A i un plan care trece
prin mijlocul M al ipotenuzei i este perpendicular pe mediana AM.
Demonstrai c exist astfel nct pentru orice punct
P s avem :
2 2 2
PC PB PA + = .
Prof. Vasile Huza , Coronini
w
w
w
.
n
e
u
t
r
i
n
o
.
r
o
45
Soluie: Cu teorema lui Pitagora i teorema medianei se ajunge destul de
rapid la 2 = .
VIII.041 Un paralelipiped dreptunghic cu dimensiunile
( ] 1 , 0 , , c b a are aria total 2s . Demonstrai c :
x s
a
x c
c
x b
b
x a
+
+
+
+
+
+
3 , x
+
.
Prof. Dumitru Btineu-Giurgiu , Bucureti
Soluie:
3
1 1 1
( ) 3
a b c a b c
E x x
b c a a b c b c a

= + + + + + +


1
( ) 3 3
x
ab bc ca sx sx
abc abc
+ + + = + + .
VIII.042 Fie a , b , c astfel nct :
1
2 2 2
= + + + + bc ab c b a .Artai c : 2 b .
Prof. Mirela Genoiu , Craiova
Soluie:
2 2
2
2 2 2
1
2 2 2
b b b
a b c ab bc a c

+ + + + = + + + + =


conduce
la :
2
1
2
b
;concluzia e imediat.
VIII.043 Demonstrai inegalitatea :

2
1
3 2 3 2 3 2

+ +
+
+ +
+
+ + y x z
z
x z y
y
z y x
x

Prof. Liviu Smarandache , Craiova
Soluie: Se noteaz numitorii cu a, b ,c se determin x , y , z n funcie de
a,b,c i se folosete
3
3
a b c a b c
b c a b c a
+ + .
VIII.044 O cldire are 72 de etaje i este dotat cu un lift special. Dac se
apas butonul galben liftul urc 7 etaje , iar dac se apas butonul verde
liftul coboar 9 etaje ( dac una din aceste comenzi nu e realizabil liftul
rmne pe loc ) . Putem ajunge cu acest lift de la etajul 1 la etajul 72 ?
Concurs Bulgaria
Soluie: Dac urcm de n ori i coborm de m ori , ajungnd la etajul
72,avem: 72 1 7 9 n m = + ( 1) , unde , n mN.Deducem c 1 * 7n e
multiplu de 9,deci n poate fi : 5,14, Dac 5 n = ,ecuaia (1) nu are
46
soluii naturale.Dac 14 3 n m = = ,deci problema are soluii.(Pentru a
nu bloca liftul procedm astfel:apsm de dou ori pe butonul
gakben,apoi o dat pe cel verde,din nou galben,apoi verde,galben,verde i
pn la sfrit de zece ori galben ! ).
VIII.045 a ) Artai c pentru orice x , y > 0 avem :

2 2 3 3
xy y x y x + + ;
b ) Dac x , y , z > 0 , demonstrai c :

2
2
2
2
2
2
3
x
z
z
y
y
x
z
x y
y
z x
x
y z
+ +

+ .
Prof. Marius Damian , Brila
Soluie: a) Avem:
( ) ( ) ( ) ( )
2
3 3 2 2 2 2
0 0, x y x y x y x x y y x y x y x y + + +

inegalitate evident.b) mprind prin
2
x y inegalitatea demonstrat mai
sus, aceasta se scrie:
2
2
1
x y x
y x y
+ +
i echivalent:
2
2
1 .
x y x
y x y
+
Au loc i
inegalitile analoage:
2
2
1 ,
y z y
z y z
+
2
2
1 .
z x z
x z x
+ Prin sumarea
ultimelor trei inegaliti, se obine concluzia.
VIII.046 Numrul x are 4n cifre , toate egale cu 3 , numrul y are 2n
cifre , toate egale cu 9 . Determinai cel mai mic numr natural z pentru
care numrul z y x A + + = 2 3 este ptrat perfect pentru orice n
*
.
Prof.Marian Bdoi , Oravia
Soluie:

( )
4
4 1
4n ori
10 1
33...3 3 10 ... 1 3
9
n
n
x


= = + + =
,

2
2n ori
10 1
99...9 9
9
n
y

= =
,
deci
( )
2
4 2 2
10 2 10 3 10 1 4
n n n
A z z = + + + = + + , aadar 4. z =
VIII.047 . Determinai numerele raionale x i y pentru care exist
m , n astfel nct m y x = + i n
y x
= +
1 1
.
Prof. Nicolae Dragomir , Prof. Tudor Deaconu , Reia
Soluie: x , y a , b , c , d cu
d
c
y
b
a
x = = , ( fracii
ireductibile ) ; cum
bd
bc ad
y x
+
= + , avem c b / d i d / b
w
w
w
.
n
e
u
t
r
i
n
o
.
r
o
47
d b = ; analog se obine , din n
y x
= +
1 1
, c a = .
Aadar y x = ; cum 1 + y x , rmne posibil doar x = y .
Din x = y = 2x =
b
a 2
i
a
b
x
2 2
= ( numere naturale ) , deducem c a
i b sunt divizori ai lui 2 i 0 >
b
a
1
1
1
= =
b
a
sau
2
1
=
b
a
sau
2
1
2
= =
b
a
, de unde obinem : x = y = 1 sau x = y = 2 sau x = y =
2
1

Variant : Ecuaia cu rdcinile x , y este
0
2
= + m mnt nt i impunem condiia ca discriminantul ei s fie ptrat
perfect . Sigur merge i aa .
VIII.048 Determinai numerele naturale nenule n pentru care exist
numerele naturale
n
x x x ,..., ,
2 1
astfel nct
n x x x
n
3 ...
2 1
= + + + i
n x x x
n
4
1
1
1
...
1 1
2 1
+ = + + +
.
Prof. Lucian Dragomir , Oelu-Rou
Soluie: Folosind inegalitatea mediilor

=
=

n
k k
n
k
k
x
n
n
x
1
1
1
, egalitile din
enun conduc la
n
n
n
n
n
1 4
3
+

de unde 3 12 4
2
+ n n ; cum n
*
,
deducem { } 3 , 2 , 1 n .Acum trebuie s vedem dac exist
k
x N ! .
Pentru n = 1 avem imediat 3
1
= x i
4
5 1
1
=
x
, absurd .
Pentru n = 2 obinem

= +
= +
8
9 1 1
6
2 1
2 1
x x
x x
; imediat avem c acest sistem nu
are soluii naturale ;
48
Pentru n = 3 , avem :

= + +
= + +
12
13 1 1 1
9
3 2 1
3 2 1
x x x
x x x

Presupunem
3 2 1
x x x i vom avea
1
3x
3 3 2 1
3x x x x + +
3
1
x , 3
3
x
Se analizeaz rapid fiecare caz n parte ( 1
1
= x nu conduce la soluii
naturale pentru celelalte dou numere )
Dac 2
1
= x , avem imediat 3
2
= x i 4
3
= x .
Deci n = 3 ,
4 , 3 , 2
3 2 1
= = = x x x
( sau permutri ) .
Clasa a IX-a
IX. 034 Fie ABC un triunghi i AD , BE , CF trei ceviene concurente n
P. Determinai poziia punctului P dac 12
2
2
2
2
2
2
= + +
PF
PC
PE
PB
PD
PA
.
Prof. Nicolae Stniloiu , Boca
Soluie: Notm , ,
PA PB PC
a b c
PD PE PF
= = = i folosim
2 2 2 2
1
( )
3
a b c a b c + + + + .Mai folosim relaiile lui Van Aubel i
2
a b
b a
+ .Deducem c cevienele sunt mediane,deci P este centrul de
greutate al triunghiului.
IX.035 Demonstrai c :

2 2
) ( 2 y x xy y x + + , x , y
+
*

Prof. Dumitru Btineu-Giurgiu , Bucureti
Soluie: Notm
2 2
x
t x yt
y
= = ;nlocuim i ,cu grij,vom ajunge la
inegalitatea echivalent
4
( 1) 0 t .
IX.036 Demonstrai c un patrulater ABCD convex i ortodiagonal este
romb dac i numai dac
AD BC CD AB CD AB BD AC + + + = +
2 2 2 2
.
Prof. Daniel Jinga , Piteti
Idee: Notm { } AC BD O = i ne jucm cu teorema lui Pitagora.
w
w
w
.
n
e
u
t
r
i
n
o
.
r
o
49
IX.037 Determinai funciile f : care verific :
) ( ) ( ) ( ) ( ) (
4 3 2 2 3 4
t f z f y f x f t z y x f + + + = + + + , x,y,z,t .
Anca Tuescu , elev , Craiova
Soluie: Facem 0 (0) 0 x y z t f = = = = = ;dac 0 y z = = ,atunci
avem:
4 4
( ) ( ) ( ), , f x t f x f t x t + = + R;pentru
4 16
( ) ( ) 0, t x f x f x x = + = R ,aadar
16
( ) ( ), f x f x x = R ( 1 ) . Pentru
4
0 ( ) ( ), y z t f x f x x = = = = R ,de unde
16 4 4 4
( ) (( ) ) ( ) ( ) f x f x f x f x = = = ( 2 ) .
Din (1) i (2) deducem ( ) ( ), f x f x x = R,adic f este funcia
constant nul.
IX.038 Rezolvai ecuaia :

2
1
2
1
4
3
4
2
4
1
+

+
=

+
+

+
+

+ x x x x x
.
Drago Ungura , elev , Oelu-Rou
Soluie: Folosim identitatea Hermite
[ ] [ ]
1 2 3
4 ,
4 4 4
t t t t t t

+ + + + + + =


R
.Notm 4 x t = i ecuaia
dat devine
1 2 3 1 4 1
2
4 4 4 2 2
t
t t t t

+ + + + + = + +


;deducem c
exist
4 1 2 1
,
2 4
t k
k k t
+
= = Z i, folosind identitatea Hermite,
[ ] [ ]
1
4 2
2
t t t k

= + +


sau [ ]
2 1 2 1
2 1 2 1 0
4 4
k k
k k
+ +
+ = + =


.
Imediat avem { } { }
1 3
0,1 , 1, 3 .
4 4
k t x





IX.039 Determinai cte elemente are mulimea
{ x / 1 2 ) 3 ( ) 1 (
2
+ = + + x x x x } .
Prof. Lucian Dragomir , Oelu-Rou
Idee:
2 2 2
( 1) ( 3) ( )( 3 ) x x x x x x x + + = + + i notm
2
2 y x x = + .
50
IX.040 Demonstrai c exist o ecuaie de gradul al doilea cu coeficieni
ntregi care are o rdcin egal cu
0
5 , 22 ctg .
Concurs Ungaria
Soluie: Se arat imediat c 1 2
8
ctg

= + ( am prefera s primim o
soluie geometric a acestui rezultat !! ) ; considerm atunci 1 2 ca
fiind cealalt rdcin i obinem,de exemplu,ecuaia
2
2 1 0 x x = .
IX.041 Demonstrai c o progresie aritmetic infinit i neconstant nu
poate avea toi termenii numere prime.
Concurs Ungaria
Soluie: Dac r este raia progresiei i
k
a un termen oarecare al
acesteia,avem :
k m k
a a mr
+
= + ;raia r este un numr natural , primul
termen este cel puin 1 , deci termenii urmtori sunt mai mari ca 1.Dac
1
k
a > i
k
m a = ,atunci (1 )
k m k k k
a a a r a r
+
= + = + este un numr
compus.
IX.042 n paralelogramul ABCD o paralel la AC intersecteaz
drepteleAB si AC n punctele M i N (MAB, NBC). Paralela
1
d prin
M la BC

i paralela
2
d prin N la AB se intersecteaz in E. Notm cu P
mijlocul segmentului [MN], cu T mijlocul lui [RS], unde {R}= d
1

AC,{S} = d
2
AC.Demonstrai, prin dou metode, c punctele B,D,E,P,T
sunt coliniare.
Prof. Mircea Iucu , Reia
Soluie: n paralelogramul MBNE , P = mijlocul diagonalei [MN] deci
B,P,E coliniare (1)n paralelogramul ABCD ,T = mijlocul diagonalei
[AC] deci B,D,T coliniare. (2)
MNSR este trapez unde B,P,E sunt coliniare.(intersectia diagonalelor i
mijloacelor laturilor paralele sunt coliniare) (3).
Din (1),(2),(3) rezulta B,D,E,P,T sunt coliniare.
Varianta 2 reprezint o metod vectorial pe care,din lipsa spaiului,nu o
vom prezenta aici.
IX.043 Rezolvai n mulimea
+
sistemul de ecuaii :

=
+

+
+

+
+

= + +
2
3
1
1
1
1
1
1
1
2
2
2
2 2
z
z
y
y
x
x
zx yz xy
x

Prof. Aurel Doboan , Lugoj
w
w
w
.
n
e
u
t
r
i
n
o
.
r
o
51
Soluie : Deoarece , , 0 , , 0,
2
x y z a b c

>


aa nct :
, , , , ... 1
2 2 2
a b c
x tg y tg z tg a b c xy yz zx = = = + + = + + = = .Folosim
apoi
2 2 2
2 2
1 1 1
1 1 1
x
x y z
x y z

+ +
+ + +
3
cos cos cos
2
a b c = + + = . Se tie ns c
ntr-un tringhi avem
3
cos cos cos
2
a b c + + ,cu egalitate doar dac
3
a b c

= = = .Obinem imediat soluia sistemului :
1 1 1
, ,
3 3 3




Clasa a X-a
X. 034 Demonstrai c pentru orice x 0 i n , n 2 , este
adevrat inegalitatea :
1 2
2
) 2 (
1

+
+

=
n
n
k
k
kx
x

Prof. Dorin Mrghidanu , Corabia
Soluie i generalizare : Dac a , x 0 , folosind formula binomului
lui Newton , avem :
(a + x)
k
= a
k
+ k . a
k-1 .
x + (o valoare pozitiv) a
k-1
. (a + k . x) ,
cu egalitate pentru k = 1 , sau cnd x = 0 si ( ) k N .
Deci ,
k x a
x a
k
.
) (
+
+

1 k
a i prin urmare ,

=
+
+
n
k
k
k x a
x a
1
.
) (

n
k
k
a
1
1
=
1
1

a
a
n

Pentru a = 2 , se obtine inegalitatea dinenun .
X.035 Determinai numerele naturale n pentru care dezvoltarea

n
) 5 2 (
3
+ conine exact 5 termeni raionali. * * *
Rspuns : {24, 26, 27, 28, 29, 31} n .
X.036 Dac x , y
+
i x + y = 2 , determinai valoarea maxim a
sumei
3
2 3 2
y x S + = .
Prof. Lucian Dragomir , Oelu-Rou
Soluie:
3 2 2 3 3 3
1 1
1 1 2
3 3
x x y y
x y x x y y
+ + + +
+ = + + =
Aadar valoarea maxim este 2 ( se obine dac 1) x y = = .
52
X.037 Dac a , b , c sunt numere reale strict pozitive cu 1 = + + c b a ,
demonstrai c : 8
1
1
1
1
1
1

+
c
c
b
b
a
a
.
Gazeta Matematic
Soluie: Aplicm inegalitatea lui Jensen funciei convexe
( )
1 2
: 1,1 , ( ) ln ln 1
1 1
x
f f x
x x
+
= =



R .
X.038 Demonstrai c n scrierea numrului
1981
) 50 2501 ( + ca numr
zecimal primele 3962 cifre dup virgul sunt zerouri.
Prof. D.M.Btineu-Giurgiu , Bucureti
Soluie:
1981 1981 1981 1981
( 2501 50) ( 2501 50) ( 2501 50) ( 2501 50) A= + = + + =
1981 *
1981
1
( 2501 50) ,
( 2501 50)
B B B = + = +
+
N Obinem c
( )
1981 1981
1981 3962
3962
1 1
, , 0,1 , 0
(50 50) ( 2501 50)
1 1
0, 00...01.
100 10
A B C C C C
C
= = < < =
+ +
= = <
_

X.039 Fie f : A B . Artai c f este injectiv dac i numai dac
A Y X , ) ( cu ) ( ) ( Y f X f Y X .
* * *
Soluie: (1) Fie A Y X , ) ( oarecare cu ) ( ) ( Y f X f ;pentru orice
( ) ( ) ( ) a X f a f X f Y b Y astfel nct
( ) ( ) f a f b a b = = (deoarece f este injectiv),deci . a Y . X Y
(2)Reciproc,fie , , ( ) ( ) a b X f a f b = .Din
({ }) ({ }) { } { } f a f b a b a b = ,adic f este injectiv.
X.040 S se determine funcia f : care satisface :
) ( ) ( ) ( y f x f y x f = + , x , y i
2006
3 ) 2006 ( = f
Prof. D.M.Btineu-Giurgiu , Bucureti
Soluie: Prin inducie(eventual)avem: ( ) 3
n
f n =
w
w
w
.
n
e
u
t
r
i
n
o
.
r
o
53
X.041 a ) Calculai suma
1
n
k
n
k
S k
C
=
=

, n
*
;
b ) Demonstrai c : ) 1 2 )( 1 ( 2 +
n n
n n , n 1 ;
c ) Demonstrai c : ) 1 2 ( 2 3
n n n
n , n 1 .
Cosmin Istodor , elev , Oelu-Rou
Soluie: a)Folosim
1
1 1 1
( 1)
2 , 2 1,
2
n n n
k n k n
n n
n n
k C n C k

+
= = =

i
inegalitatea lui Cebev:
1 1 1
( )( )
n n n
k k
n n
n k C k C

;
b) din nou Cebev :
0 0 0
( 2 ) ( )( 2 )
n n n
k k k k
n n
n C C

.
X.042 S se arate c dac z i 1 < z , 1 1 < + z , atunci
4 1
3
< + z .
Prof. Aurel Doboan , Lugoj
Soluie :
3 3 3
1 ( 1) 3 ( 1) ( 1) 3 ( 1) z z z z z z z + = + + + + + =
3
1 3 1 4. z z z = + + + <
X.043 Rezolvai ecuaia :
x x x x
255 64 225 136 + = + .
Prof. Aurel Doboan , Lugoj
Soluie : Calcule destul de rapide conduc la :
( )( )
15 8 15 8 17 0
x x x x x
+ = .
Obinem soluiile 0, 2 x x = = (unicitatea celei de a doua se obine
folosind monotonia funciei
8 15
: , ( )
17 17
x x
f f x

= +


R R ) .
Clasa a XI-a
XI. 034 Dac f : [ a , b ] este o funcie derivabil cu proprietatea c
f( a ) = 0 , demonstrai c exist c ( a , b ) astfel nct
) ( ) ( ) (
/
c f c b c f = .
Prof. Dorin Mrghidanu , Corabia
Soluie : Considerm funcia :[ , ] g a b R , g ( x ) = ( b x ) f(x),
care este de asemenea derivabil i n plus g ( a ) = g ( b ) = 0 .
54
Cu teorema lui Rolle deducem c exist c ( a , b ) , astfel nct
g( c ) = 0 . Cum g(x) = - f ( x ) + ( b x ). f

( x ) , rezult c
f ( c ) + ( b c ).f

( c ) = 0 .
XI.035 Fie f : o funcie continu , mrginit , cu 0 ) 0 ( f .
Artai c exist x
1
, x
2
,
2 1
x x , astfel nct :
0 ) ( ) (
1 2 2 1
= + x f x x f x .
Prof. Lucian Dragomir , Oelu Rou
Soluie : f mrginit
, , ( ) , m M m f x M x R R .Considerm funciile continue
, : , ( ) ( ) , ( ) ( ) g h g x f x x h x f x x = = + R R i din
( ) ( ) 0, ( ) ( ) 0 g m g M h m h M deducem c
[ ]
1 2
, , x x m M astfel
nct
1 2
( ) 0, ( ) 0 g x h x = = ,adic
1 1 2 2
( ) , ( ) f x x f x x = = .nmulim
aceste dou egaliti cu
2
x ,respectiv cu
1
x i le adunm. ( E nevoie de
ipotezele 0 ) 0 ( f i
2 1
x x ? Putei da un exemplu de funcie f care
satisface enunul ? ).
XI.036 Dac f este o funcie derivabil i cu derivata funcie
periodic , artai c f se poate scrie ca o sum dintre o funcie periodic
i o funcie de gradul nti.
Prof.Dorin Mrghidanu , Corabia
Soluie: Cum f

este periodic , () T > 0 , astfel nct
f

( x + T ) = f

( x )
Deci f( x + T ) i f( x ) difer printr-o constant k R , adic ,
( ) ( ) f x T f x k + = (1)
Fie funcia : , ( ) ( )
k
x f x x
T
= R R (2)
Un calcul imediat conduce la ( ) ( ) 0 x T x + = , deci este
periodic de perioada T . Din (2) , avem ( ) ( )
k
f x x x
T
= + , care
rspunde cerinelor enunului .
XI.037 Calculai { }) 1 (log lim
2
2
+ +

n n arctg
n
, unde {x} reprezint
partea fracionar a lui x.
Prof. Nicoleta Bran , Craiova
w
w
w
.
n
e
u
t
r
i
n
o
.
r
o
55
Soluie :
{ }
2 2 2 2
1 1 1 1 n n n n n n n n n

+ + = + + + + = + +

;limita
este
2
2
1
lim log
4
1
n
n
L arctg
n n n

+
= =
+ + +
.
XI. 038 Fie f o funcie care are proprietatea lui Darboux i care
satisface x
x
x f
f =

) (
, x > 0 . Demonstrai c f este continu .
Prof. Ionu Ivnescu , Craiova
Soluie : Considerm
( )
( )
f x
g x
x
= i se obine imediat c
( ( )) , 0 g g x x x = > ,adic 1 g g =
R
;deducem c g este injectiv,apoi
continu , de unde f este continu.
XI.039 Determinai funciile continue f : care satisfac :

xy
xy f
x f
x
y
y f
y
x ) ( 2
) ( ) ( = + , x , y
*
.
Prof. Lucian Dragomir , Oelu-Rou
Soluie : Facem
2 2 *
( ) ( ), x y x f x f x x = = R .Facem succesiv
transformarea
1
2
x x i nmulim egalitile obinute,deducem astfel :
2 1
1 1 1 1
1 ...
2 2 2 2
( ) ( )
n n
x f x f x
+
+ + + +
= .Trecem la limit pentru n ,folosim
continuitatea lui f i ajungem la :
2
( ) , (1) f x ax a f = = R.
XI.040 Determinai n
*
i matricele inversabile A M
n
() pentru
care
* * *
) ( A A = , unde A
*
este adjuncta matricei A.
Prof. Romana i Ioan Ghi , Blaj
Rspuns: Dac det d A = ,se deduce : { }
2
1 2
( ) 1, 2
n n n
d d n

= .
Verificri simple conduc la
*
0
,
0
a
A a
a

=


C .
XI.041 Se consider un ptrat cu latura de lungime 1 . Ducem cte dou
drepte paralele la laturile ptratului i acesta se mparte n 9 ptrate
congruente;eliminm ptratul din mijloc.Cele 8 ptrate rmase le
mprim n mod analog n cte 9 ptrate i din nou din fiecare eliminm
ptrelul din mijloc ,etc. Repetm procedeul de n ori.
56
a ) Cte ptrate cu latura de lungime
n
3
1
rmn ?
b) Dac notm cu S ( n ) aria total a ptratelor eliminate , calculai
) ( lim n S
n
.
Concurs Ungaria
Soluie: a) La prima mprire , eliminnd un ptrat din cele 9
obinute,rmn 8 ptrate;dup a doua mprire,eliminnd 8 ptrate din
cele 8 9 obinute,rmn
2
8 ptrate.Continund procedeul,dup n
mpriri,rmn 8
n
ptrate;latura fiecruia va fi
1
3
n
; b) aria total a
ptratelor rmase este
2
1 8
8
3 9
n
n
n

=


i astfel
8
( ) 1
9
n
S n

=


;evident ,limita cerut este egal cu 1 .
XI.042 a ) Artai c pentru orice matrice A , B M
2
( ) are loc
relaia : ) det (det 2 ) _ det( ) det( B A B A B A + = + + ;
b ) Dac X , Y M
2
( ) astfel nct

2
1
) det( ) det(
2 2
= + + + YX XY Y X ,
atunci 2 ) det( ) det( + + Y X Y X .
Prof. Marius Damian , Brila
Soluie: a) Fie funcia polinomial
( ) ( ) : , det . f f x A xB = + R R
Exist k R astfel nct
( )
2
det det . f x A kx x B = + +
Atunci
( ) ( ) ( ) ( ) ( ) 1 1 det det 2 det det . f f A B A B A B + = + + = +
b) n egalitatea de la punctul precedent facem
2 2
, A X Y B XY YX = + = + i obinem
( ) ( ) ( ) ( )
2 2
2 2
det det 2 det det . X Y X Y X Y XY YX

+ + = + + +



De aici, innd cont de ipotez i de inegalitatea Cauchy - Schwarz, putem
scrie
w
w
w
.
n
e
u
t
r
i
n
o
.
r
o
57
( ) ( ) ( ) ( )
2 2 2
det det 2 det det X Y X Y X Y X Y + + + + =


( ) ( )
2 2
4 det det 2 X Y XY YX

= + + + =

i concluzia rezult imediat.
XI.043 Se consider irul definit prin
5
26
1
= x ,
4 12 13 5
2
1
+ =
+ n n n
x x x , n 1 . Calculai
n
n
n
n
x


5
lim .
Prof. Aurel Doboan , Lugoj
Soluie:
1
5
5
n
n n
x = + (inducie) i limita cerut este 1 .
XI.044 a) Calculai
12
sin

;
b) Fiind date matricele


=
1 3
3 1
X ,

=
3 1
1 3
Y ,calculai
( )
n
Y X
2
+ , n
*.

Prof. Aurel Doboan , Lugoj
Soluie:
cos sin
2 2
,
sin cos 4 4 12
X Y



+ = =


;se demonstreaz
inductiv :
cos sin cos sin
sin cos sin cos
n
t t nt nt
t t nt nt

=


i astfel matricea
cerut este egal cu :
3 *
cos sin
2 , ,
sin cos 6
n
u u
n
u n
u u


=


N .
Clasa a XII-a
XII. 034 Fie a , b > 0 , a < b i funcia continu f : [ a , b ]
+
*
. Dac

=
+ +
b
a
b
a
dx x f dx
x b a f x f
x f
) (
) ( ) (
) (
, artai c exist c ( a , b )
astfel nct c c f b a = + ) ( ) ( .
Prof. Dumitru Btineu-Giurgiu , Bucureti
58
Soluie: Facem schimbarea de variabil destul de bine cunoscut
x a b t = + .Notnd cu I integrala din dreapta vom ajunge la
2
b a
I

=
i ,folosind ipoteza:
1
b
a
I xdx
a b
=
+

.Acum ajungem la :
( ( ) ) 0
b
a
x
f x dx
a b
=
+

.Aplicai teorema de medie !.


XII.035 Fie ( , G ) un grup i a , b G , e b a , , a , b distincte . Dac
4 2
b ab = i
2 5
a b = ,determinai ordinele elementelor a i b .

Prof. Romana i Ioan Ghi , Blaj
Soluie: nmuliri la stnga i la dreapta cu tot felul de puteri ( trebuie s
avei rbdare i nervi) conduc la
2
; ( ) 2 b e b e ord b = = .Din
2 5
a b = deducem
4
( ) 4 a e ord a = = (nu se poate mai mic;ar putea fi
doar 2 , dar asta ar conduce la
5
b e = i mai departe la o contradicie.
XII.036 Calculai
n
n
n n
n
n
n n
n
2
1 2
sin ...
2
3
sin
2
1
sin
2
2
sin ...
2
4
sin
2
2
sin
lim

+ + +
+ + +


Prof. Viorel Cornea , Dan tefan Marinescu , Hunedoara
Soluie: Notm
n
a =
2 4 2
sin sin ... sin
2 2 2
n
n n n
+ + +
i
n
b =
1 3 2 1
sin sin ... sin
2 2 2
n
n n n

+ + + ;
1
0
1 1
0 0
1
sin
2
lim 1
1
sin sin
2
n
n
n
xdx
a
b
xdx xdx

= =



XII.037 Fie G un grup multiplicativ i a , b G astfel nct
10 10
b a = i
17 17
b a = . Demonstrai c a = b .
Prof. Aurel Doboan , Lugoj
Soluie : Din
17 17
a b = deducem
7 7
a b = ,apoi din
10 10
b a = avem
3 3
a b = ,mai departe
7 7
a b = conduce la
4 4
a b = ,de unde . a b =

w
w
w
.
n
e
u
t
r
i
n
o
.
r
o
59
XII.038 Artai c exist q astfel nct :
e q dx e dx x
x
e
= +

2
0
sin
1
) arcsin(ln .
Prof. Aurel Doboan , Lugoj
Soluie : Putem (i altfel) folosi egalitatea lui Young : dac
[ ] [ ]
: , , f a b c d e continu i bijectiv , atunci
1
( ) ( )
b d
a c
f x dx f x dx bd ac

+ =

.E suficient s alegem
[ ] : 1, 0, , ( ) arcsin(ln )
2
f e f x x

=


.
XII.039 Care dintre urmtoarele numere este mai mare :

=
3
2
arctgxdx a sau

=
2
3
arctgxdx b ?
Gazeta Matematic
Soluie:
3 2 3
2 2 2
a arctgxdx arctgxdx arctgxdx A B

= = + = +

i
2 2 2
3 3 2
b arctgxdx arctgxdx arctgxdx C A


= = + = +

;deoarece
0, 0 arctgx x < < i 0, 0 arctgx x > > . B C a b > >
XII. 040 Exist funcii f : continue care admit o primitiv F
pentru care 0 )) ( (
3
= + + x x x F f , x ?
Prof. Antoanela Buzescu , Caransebe
Soluie: Presupunem c exist funcii cu proprietatea din enun ;
g : , x x x g =
3
) ( este strict cresctoare , deci injectiv
g F f = injectiv F injectiv ; F derivabil F continu i deci
F este strict monoton f F =
'
pstreaz semn constant pe ( * )
0 2 )) 1 ( ( > = F f i 0 2 )) 1 ( ( < = F f contradicie cu ( * ) .

60
XII.041 Calculai : dx
x x tg
e
I
x


=
sin
2
,

2
, 0

x
Prof. Nicolae Dragomir , Reia ,Prof.CarmenDragomir,Timioara
Soluie:

=

= dx
x
e
dx e
x
x
dx e
x x
x
I
x
x x
sin 2
1
sin
cos
2
1
sin
1
sin
cos
2
1
'
2
'
2

=

+

dx
x
e
e
x
x
dx
x
x x e
e
x
x
x
x
x
x
'
2 2 2
sin 2
1
sin 2
cos
sin
) sin (cos
2
1
sin
cos
2
1
= C
x x
x e
x
+

+
sin
1
sin
cos
2
2
.
XII. 042 Se consider mulimea de matrice M =

+
+
5
1
, /
1 3 0 6
0 0 0
0 1 2
x x
x x
x x
i notm G = {
5
1
} . Dac
pentru orice x , y notm xy y x y x 5 + + = , artai c M are o
structur de grup n raport cu nmulirea matricelor , izomorf cu grupul
( G , ) .
Prof. Nicolae Dragomir , Prof. Tudor Deaconu , Reia
Soluie : Notm cu M ( x ) o matrice din mulimea dat i avem
xA E x M + = ) ( , unde

=
1 0 0
0 0 0
0 0 1
E i

=
3 0 6
0 0 0
1 0 2
A ; calcule
simple : A A A AE EA E E 5 , ,
2 2
= = = =
) 5 ( ) ( ) ( xy y x M y M x M + + = M ; n plus , se arat c
5
1
5 + + xy y x , x , y
5
1
; verificarea axiomelor grupului
devine astfel imediat , elementul neutru este M ( 0 ) , etc.
Funcia f : G M , f ( x ) = M ( x ) este bijectiv i verific condiia de
morfism .
w
w
w
.
n
e
u
t
r
i
n
o
.
r
o
61
XII.043 Fie ) , ( G un grup multiplicativ avnd elementul neutru e .
Artai c dac exist un morfism injectiv f : G G pentru care avem
e x f x f f = ) ( )) ( ( , x G , atunci G este abelian .
Prof. Lucian Dragomir ,Oelu-Rou
Soluie : Din ipotez ( )
1
( ( )) ( ) f f x f x

= i nlocuind x cu f ( x )
avem ( ) ) ( ) ( )) ( ( ( ))) ( ( (
1
1 1
x f x f x f f x f f f = = =


.
Cum f este injectiv avem c x x f f = )) ( ( deci x x f =
1
)) ( ( ,
adic
1
) (

= x x f ;
Faptul c aceast funcie este morfism se scrie deci
1 1 1
) (

= y x xy ,
x ,y G , adic yx y x xy = =
1 1 1
) ( , x , y G , aadar G este
abelian .


Concursul Judeean al Revistei de Matematic
Cara-Severin , Ediia a II-a
Regulament
Ediia a II a a Concursului Revistei este n plin desfurare ,
urmeaz un nou set de probleme.Fiecare elev trebuie s rezolve
(subliniem din nou: singur ! ; altfel e posibil s v trezii calificai la
concurs i acolo s nu facei mare lucru dai natere la ntrebri i
credem c nici n-o s v simii prea bine), aadar s rezolve ct mai
multe probleme de la clasa sa , de la clasa precedent sau de la orice
clas superioar (am avut anul acesta multe situaii i de acest gen).
Redactai ngrijit fiecare problem pe cte o foaie separat (enun + autor
+ soluie + numele vostru) , completai talonul de concurs de pe ultima
pagin a revistei i trimitei totul ntr-un plic (ncercai s fie unul ceva
mai mare , format A5 cel puin -nu nghesuii tot n ceva mic ) adresat
astfel :
Prof.Lucian Dragomir , Grup colar Industrial Oelu-Rou, str.Republicii
10-12, 325700, Oelu-Rou, Cara-Severin, cu meniunea probleme
rezolvate. Insistm asupra trimiterilor n plic (nu n folii de plastic) i
asupra respectrii cu strictee a termenelor finale indicate de fiecare dat
- plicurile primite dup data limit nu vor fi luate n considerare.
Dup data limit de trimitere a soluiilor, acestea sunt evaluate i
n numrulul urmtor al revistei vor fi publicai toi rezolvatorii cu
punctajele obinute.
62
La ediia a II-a a concursului vor fi selectai concurenii n funcie de
punctajele obinute din rezolvarea problemelor publicate n numerele 15,
16 , 17 i 18 ale revistei noastre. n jurul datei de 20 ianuarie 2007 se va
ntocmi clasamentul general (prin nsumarea punctelor obinute) i astfel
primii clasai(n jur de 10 de clas i care au minim din jumtatea
punctajului maxim posibil) vor fi invitai, mpreun, ca i la ediia
precedent, s participe la concurs; acesta va avea loc tot la nceputul
lunii februarie ntr-un ora care va fi anunat n timp util .
Subiectele vor fi alese tot din probleme de genul RMCS sau G.M. sau
ceva ct de ct nou. Noutatea ediiei din acest an const n faptul c ne
adresm de-acum i elevilor de ciclu primar.
Demarm cu aceast ocazie i un concurs (cu premii din nou) de
probleme propuse de ctre elevi; acestea trebuie trimise n plic separat de
eventualele probleme rezolvate, cu meniunea Probleme Propuse .
ncercai ! Oricum , n-o s v par ru .
Spor la treab tuturor : elevi , profesori , prini sau prieteni !
(Informaii suplimentare se pot obine la : prof. Lucian Dragomir ,
tel: 0255/530303 sau 0722/883537).

Not : Rugm toi colaboratorii care ne trimit probleme propuse
( obligatoriu cu soluii ! ) , note, articole, etc., s tehnoredacteze
materialele pe calculator i s le ataeze ca fiier la mesajul lor, apoi s
foloseasc adresa : lucidrag@yahoo.com ( cu meniunea : materiale
pentru RMCS ) .
Probleme propuse
( pentru participare la concurs i nu numai )

(Data limit de trimitere a soluiilor : 30 octombrie 2006)

Clasa a IV-a

IV.025 ntr-o cuc se afl iepuri de cas i fazani , n total 100 de
picioare i 36 de capete. Ci fazani i ci iepuri de cas sunt n cuc ?
Inst.Ozana Scrin , Reia
IV.026. 6 muncitori execut 288 piese n 4 ore. Cte piese vor executa 10
muncitori n 5 ore ?
Inst.Ozana Scrin , Reia
w
w
w
.
n
e
u
t
r
i
n
o
.
r
o
63
IV.027. Dan i Alina au vndut unui centru de achiziii a fructelor 166 de
kg de cpuni.Cte kg a vndut fiecare dac Dan a vndut cu 6 kg mai
mult dect Alina ?
Inst.Ozana Scrin , Reia
IV.028. Suma a dou numere este 40,iar diferena lor este dublul celui
mai mic dintre ele.Aflai numerele.
* * *
IV.029. Lucia i Maria au mpreun 16 mere.Dac Lucia are de 3 ori mai
multe mere dect Maria,cte mere are fiecare ?
* * *
IV.030. M gndesc la un numr.l mpart la 2;noul numr l mpart la 3i
observ c am obinut un numr cu 35 mai mic dect cel de la
nceput.Putei gsi numrul la care m-am gndit ?
* * *
IV.031. Andrei,Bebe i Costel colecioneaz mainue.Andrei are de dou
ori mai multe dect Bebe,iar Costel de trei ori mai puine dect Bebe.Cte
mainue are fiecare dac mpreun au 50 de buci ?
* * *
IV.032. n cte feluri se poate completa un tabel
cu numerele 1,2,3,respectnd regulile :
1)pe fiecare linie s avem toate numerele;
2)suma numerelor de pe fiecare linie s fie aceeai
3)suma numerelor de pe fiecare coloan s fie aceeai ?

* * *
IV.033. Un tren circul,cu aceeai vitez,ntre localitile A i D,trecnd
prin oraele B i C.

A B C D
Se tie c distana dintre A i B e parcurs ntr-o or,distana dintre B i C
n jumtate de or,iar distana dintre C i D n dou ore.Dac ntre B i D
sunt 250 km , care este distana dintre A i C ?
* * *
IV.034. Athos,Porthos i Aramis au nvins n dueluri 140 de
dumani.Dac ar fi nvins cu 10 lupttori mai mult,Porthos ar fi nvins tot
atia ct Athos,iar dac ar fi nvins cu 20 mai puin,ar fi cigat tot attea
dueluri ca i Aramis.Ci inamici a nvins fiecare dintre cei trei
muchetari ?
* * *


64
IV.035. Dintre Anca,Bianca,Cristina,Dorel,Elena,Florin i
Gheorghi,profesorul de sport trebuie s aleag o echip format din
dou fete i un biat.n cte feluri poate face alegerea ?
* * *
IV.036. Bunicul lui Mihai are o livad cu peste 90 de pomi.O treime sunt
pruni,un sfert sunt cirei,iar restul sunt meri.Ci pomi sunt n livad
tiind c sunt mai puin de 100 ?
* * *
IV.037. Tatl lui Mihai este zidar.Pentru a construi un zid el dispune de
240 de crmizi de form paralelipipedic cu dimensiunile de 5 cm , 8 cm
i 25 cm. Dac grosimea zidului trebuie s fie de 8 cm i lungimea de 1 m
, poate fi zidul mai nalt dect Mihai ?
* * *
IV.038. n campionatul de fotbal al Romniei se acord 3 puncte echipei
care ctig meciul,echipa care pierde nu primete nici un punct , iar n
caz de egalitate fiecare echip primete cte un punct.Echipa favorit a lui
Sergiu este Poli Timioara ( normal,e bnean);dac dup 27 de meciuri ,
Poli are 51 de puncte , cte meciuri a pierdut , tiind c 9 meciuri a
ncheiat la egalitate ?
* * *
IV.039. Sergiu,Sorin i Costel se ntrec n fiecare zi la matematic.Luni,e
rndul lui Costel s le propun o problem: Dac numrului 84 i asociez
numrul 12, iar numrului 42 i asociez 6 , ce numr asociez lui 52 ?
Dup un minut (gndesc rapid bieii) , Sergiu zice : 7 , iar Sorin : 8.Cine
a ctigat concursul de luni ?

IV.040. Gheorghi pzea pe un ima civa berbecui i nite bobocei de
gsc. ntrebat de veriorul su Ionu ci berbecui i ci bobocei are, el
a rspuns: - Sunt 26 capete i 70 picioare.
Ajutai-l pe Ionu s gseasc rspunsul la ntrebare.
Inst. Mariana Mitric, c. Nr. 9, Reia
IV.041. Scriei numrul 1092 ca o sum de trei termeni astfel nct fiecare
termen s fie dublul precedentului.
Inst. Mariana Mitric, c. Nr. 9, Reia
IV.042. Se consider irul de numere : 1 ; 6 ; 11 ; 16 ; ..
a ) Completai irul cu nc doi termeni ;
b ) Gasii al 100-lea termen.
nv. Ana Modoran,Reia

w
w
w
.
n
e
u
t
r
i
n
o
.
r
o
65
Clasa a V-a
V.049 Fie mulimile:
{ } { } { }
{ } { }
1 2 3
4 5
M 1 ; M 1;3 ; M 1;3;6 ;
M 1;3;6;10 ; M 1;3;6;10;15 ;.....


= = =
= =

a) Artai c exist N
*
k, p astfel nct 55
k p
M M .
b) Exist N
*
t astfel nct
2006 ?
t
M

c) Aflai numrul elementelor divizibile cu 5 din
2006
. M
Prof. Nicolae Stnic, Brila
V.050 Se d irul de numere: 1; 1; 2; 5; 12; 27; 58;
a) Completai irul cu urmtorii trei termeni.
b) Calculai suma primilor 100 de termeni ai irului.
Prof. Marius Damian, Brila
V.051 Se consider irul de numere naturale 1, 2, 2, 3, 3, 3, 4, 4, 4, 4, 5, 5,
5, 5, 5, , unde fiecare numr natural nenul n apare de n ori.
a) S se determine al 2006-lea numr care apare n acest ir.
b) Notnd cu S suma primelor 2006 numere din acest ir, s se
studieze dac S+3 poate fi ptrat perfect.
Problem selectat i prelucrat de Prof. Laura Marin , Galai
V.052 Pentru a i b numere naturale, a < b, se consider mulimea M(a, b)
= { } / x N a x b i se noteaz cu card(a, b) numrul elementelor
mulimii M(a, b). Dac S = card(1, 2) + card(2, 4) + card(3, 6) + +
card(1003, 2006), s se arate c S este multiplu de 17.
Prof. Viorel Ion , Galai
V.053 Artai c numrul natural
2 2006 2005 ... 3 2 1
2006 2006 2006 2006 2006
+ + + + + + nu poate fi ptrat
perfect.
Problem selectat i prelucrat de Prof. Laura Marin,Galai
V.054 Vrsta Oanei mpreun cu vrsta lui Bogdan i dublul vrstei lui
Vlad este 44 de ani.
tiind c vrsta lui Bogdan este cu 4 ani mai mic dect a Oanei,
iar vrsta lui Vlad este cu 2 ani mai mare dect a lui Bogdan, s se afle ce
vrst are fiecare.
Prof. Tudor Deaconu, Reia
V.055 Aflai suma numerelor mai mici dect 1000 care mprite la 5 dau
restul 3.
Prof. Marius andru, Reia
66
V.056 Se dau numerele
3 2 2 1
4 8 64 ,
n n n
A n
+ +
= N i
2006 2005 2004
2 2 2 , B n = N. Aflai cea mai mic valoare a lui n
pentru care B divide pe A.
Prof. Marian Bdoi, Oravia
V.057 Dac a , b, c, d sunt numere naturale astfel nct a < b < c , gsii
cte triplete ( a , b , c ) satisfac egalitatea : ddd cab bca abc = + + ?
Prof. Adriana Dragomir , Oelu-Rou
V.058 S se arate c oricare ar fi mN exist n,a,b,c N astfel nct
2 2 2
4 289 17
m n
a b c + = + +
Prof. Mariana Drghici , Reia
V.059 Care dintre numerele
a = 2005
2006
+ 2006
2005
i b = 2005
2005
+2006
2006
este mai mare ?
Prof. Mariana Drghici , Reia

Clasa a VI-a
VI.049 Aflai numrul natural , abc scris n baza 10, tiind c :
10 1 82.

+ =


ab bc
c a

Prof. Nicolae Stnic, Brila
VI.050 a) S se arate c ntre oricare dou puteri naturale consecutive ale
lui 3 se afl cel puin o putere a lui 2.
b) Exist dou puteri naturale consecutive ale lui 3 ntre care s gsim
trei puteri distincte ale lui 2?
Prof. Marius Damian, Brila
VI.051 ) Dou unghiuri complementare au msurile n grade egale cu a
i b (
*
; N b a ) direct proporionale cu numerele prime
1
p i
2
p .
Dac
1 2
a b
p p
+
+
este numr natural par, determinai msurile celor
dou unghiuri.
Prof. Cecilia Solomon,Galai
VI.052 Comparai fraciile:
614
921
2006 11
2006 5
+
+
i
2109
1406
2005 10
2005 11
+
+

Prof. Mariana Iancu, Oravia
w
w
w
.
n
e
u
t
r
i
n
o
.
r
o
67
VI.053 Se consider unghiurile adiacente AOC i BOC astfel nct
[ [ OA OB i [ ] [ ] OA OB . Fie unghiul drept COD < neadiacent
unghiului BOC < cu [ ] [ ] OC OD . Artai c: a) [ ] [ ] AC BD
b) unghiurile AOD i BOC au aceeai bisectoare.
Prof. Marius andru, Reia
VI.054 Aflai numerele abc scrise n baza 10 tiind c sunt
ndeplinite simultan condiiile :
1 ) a + b este numr prim ;
2 )
2 2 2
c b a = + ;
3 ) 2a + 6b + 9c se divide cu 3 .

Prof. Marian Bdoi , Oravia
VI.055 Trei vnztori au caiete cu acelai pre. Primul a mrit preul cu
20% i apoi l-a micorat cu acelai procent, al doilea a micorat mai nti
preul cu 20% i abia apoi l-a mrit cu acelai procent iar al treilea a lsat
preul neschimbat. De la care vnztor ai cumpra acum i de ce?
Olimpiad Vaslui 2006
VI.056 2006 puncte distincte au fost fixate pe mai multe segmente
obinndu-se 2198 de segmente.Cte segmente au fost la nceput ?
Prof.Vasile erdean , Gherla
VI.057. Fie numrul :
407
) ( ) ( 3 ba ab bba aab
N
+ +
=

a) Artai c numrul N este ptrat perfect ,unde a i b sunt cifre n
sistemul zecimal.
b) Determinai cea mai mic i cea mai mare valoare a numrului N.
c) Aflai valorile lui N care se divid cu 24.
Prof.Groza Ioan , Turda
VI.058. Determinai numerele ntregi x , y care satisfac

5
1 1
x y
y x x y
= =
+ +

Prof. Lucian Dragomir , Oelu-Rou
VI.059 Gsii un numr natural care este produs a 6 numere prime
consecutive i pentru care suma divizorilor proprii primi este 41.
Prof. Dana Emilia-Schiha,Berzasca


68
Clasa a VII-a
VII.049 Fie triunghiul ABC, I centrul cercului su nscris i
{ }. = AI BC D
O dreapt perpendicular pe dreapta AI intersecteaz
( ) AB i
( ) AC n punctele P i respectiv Q, iar M i N
( ) ( ) ( )
, M BD N DC sunt simetricele punctelor P i Q fa de dreptele
BI i respectiv CI. S se demonstreze c
[ ] [ ] MD DN
dac i numai dac
[ ] [ ]. AB AC

Prof. Marius Damian, Brila
VII.050 S se determine cel mai mic numr natural format din 30 de cifre
care are suma cifrelor 30 i se divide la 30.
Gazeta Matematic
VII.051 Fie m, n . N S se arate c dac

3n 4 5m 8
a ,
2n 3 2m 3
+ +
= +
+ +
N
atunci a=4.
Prof. Marius Damian i prof. Nicolae Stnic, Brila
VII.052 a)Artai c ptratul oricrui numr natural este de forma 3k sau
3 1, k k + N
b) Determinai numerele prime p i q, p q < astfel nct
2 2
298 p q + =
Prof. Marius Golopena, Bile Herculane
VII.053 Fie ABCD un patrulater convex cu . A C < < Dac
( ) M AC astfel nct ( ) DM i ( ) BM sunt bisectoare ale unghiurilor

ADC respectiv

ABC , se cere:
a) demonstrai c BD AC
b) bisectoarele celor patru unghiuri ale patrulaterului sunt concurente.
Prof. Irina Avrmescu, Reia
VII.054. Fie un triunghi oarecare ABC i M (BC), N (AC), P (AB)
astfel nct MN AB, PM AC i PN BC.
Demonstrai c M, N i P sunt mijloacele laturilor triunghiului ABC.
Olimpiad Vaslui 2006
VII.055 n triunghiul ABC o median este perpendicular pe o
bisectoare.Lungimile laturilor sunt trei numere naturale consecutive.
Calculai perimetrul triunghiului ABC.
Prof.Vasile erdean , Gherla
w
w
w
.
n
e
u
t
r
i
n
o
.
r
o
69
VII.056 Fie .... , 0
3 2 1
a a a scrierea zecimal a numrului
13
7
6
1
+
.
Determinai
2005
a . * * *
VII.057. Fie M i N mijloacele laturilor [DC] i respectiv [AD] ale
ptratului ABCD iar CNBM={P}.Artai c : CN BM
Prof.Simona i Cristian Pop,Cluj-Napoca
VII.058 Determinai numerele ntregi x , z care satisfac :

2 2
5 3 5 x xy y + = .
Prof. Lucian Dragomir , Oelu-Rou
VII.059 Fie triunghiul ABC i I centrul cercului su nscris. Notm cu M,
N mijloacele laturilor AB, respectiv AC i cu P, Q interseciile dreptei MN
cu dreptele BI, respectiv CI.
tiind c 10 BC cm = i 18 , AB AC cm + = s se calculeze
lungimea segmentului PQ.
Prof. Marius Damian, Brila
Clasa a VIII-a
VIII.049. Fie unghiul , <XOY cu
( )
0
= = < m XOY constant,
0 90, < < P un punct variabil n interiorul unghiului , <XOY
nesituat pe bisectoarea acestuia, M i N simetricele lui P fa de OX i OY.
Ducem
{ } , , , , , = PS OM S OM PR ON R ON PS OX H
{ }. = PR OY T

a) Demonstrai c
. | NT MH

b) Artai c
( ) ( ) + = < < m PMH m PNT
constant.
Prof. Carmen Botea, Brila
VIII.050 Fie triunghiul ABC i I centrul cercului su nscris. Notm cu M
i N mijloacele laturilor AB i respectiv AC, iar cu P i Q interseciile
dreptei MN cu dreptele BI i respectiv CI. tiind c
AI
PQ ,
2
=
s se
demonstreze c
( )
0
m PAQ 30 . = <

Prof. Marius Damian, Brila
VIII.051 S se demonstreze c ecuatia : x
4n
=y
2n-1
+z
2n+1
,
are o infinitate de soluii in mulimea numerelor naturale .
Prof. Dorin Mrghidanu, Corabia
70
VIII.052 Se consider trapezul ABCD, , . AB CD AB CD > | Fie M
mijlocul laturii AD, N mijlocul laturii AB, P mijlocul laturii CD i
{ } . Q MN CD =
a) Artai c dreptele BD i NQ sunt paralele.
b) Aflai valoarea raportului dintre aria triunghiului MNP i aria
trapezului ABCD.
Prof. Marius andru, Reia
VIII.053 Cte numere de patru cifre
1 2 3 4 1
, 5 a a a a a au proprietatea c
numrul
1 2 3 4
4 3 N a a a a = + + se divide cu 13?
Prof. Stniloiu Nicolae, Boca
VIII.054 S se determine numerele pozitive
1 2
, a a pentru care
1 2
1 a a + = i ( ) ( ) ( ) ( )
1 2 1 2
1 1 1 1 a a a a + + + este numr
natural.
prof. N. Dragomir, T.Deaconu, Reia
VIII.055 S se arate c dac triunghiul ABC este dreptunghic n A i
( ) , AD BC D BC , atunci
2
AB AC AD
BC
+ +
< .
Prof. Marius andru, Reia
VIII.056 Fie , , , x y z numere pozitive cu 1. x y z = Demonstrai
inegalitatea:
2 2 2 2 2 2
.
x y y z z x
x y z
x y y z z x
+ + +
+ + + +
+ + +

Prof. Marius Damian, Brila
VIII.057 Fie a, b, c numere reale pozitive care satisfac egalitatea:
2 2 2
1 a b c + + = . S se arate c:
3 3 3
6 a b c a b c abc + +
Stniloiu Ovidiu, elev Boca
VIII.058. Fie [ABCD] un tetraedru regulat avnd lungimea muchiei l i
AE nlimea din A a triunghiului ABC, [ ] E BC . S se calculeze
distana dintre dreptele AE i BD.
Stniloiu Ovidiu, elev Boca
VIII.059 Fie a o rdcin a ecuaiei 0 1 7
2
= + x x . S se calculeze
i
a
a
2
2
1
+
3
3
1
a
a + .
Prof. Mariana Drghici , Reia

w
w
w
.
n
e
u
t
r
i
n
o
.
r
o
71
Clasa a IX-a
IX.044 . Pentru orice funcie f : i orice numr natural m
notm A
m
= { x / f ( x ) = m } .
Spunem c o funcie f : este o funcie simpl dac pentru
orice m , mulimea A
m
are cel mult dou elemente .
a ) Demonstrai c nu exist funcii simple f cu proprietatea c :
1 ) ( ) ( + = + x x f x f x , x ;
b ) Determinai funciile simple f care satisfac :
1 ) ( ) 1 ( ) ( + = + x x f x x f x , x .
Prof. Lucian Dragomir , Oelu-Rou
IX.045 S se rezolve n R ecuaia: {x}
2007
-[x]
2007
-x
2007
=0
Prof. Carmen Botea, Brila
IX.046 Pe laturile ( AB ) , ( BC ) i ( CA ) ale triunghiului ABC se
consider punctele M ,N , respectiv Q astfel nct AM = BN = CP .
Demonstrati c dac triunghiurile ABC i MNP au acelai centru de
greutate , atunci ABC este triunghi echilateral .
Prof. Nicolae Stniloiu , Boca
IX.047 Fie a , b care satisfac
2 2 2 2
6 5 ) 1 ( 4 ) 10 ( 2 a b b a + = + + +
Artai c dac a + b 21 , atunci 2006 6 5
2 2
= + a b .
Adriana i Lucian Dragomir , Oelu-Rou
IX.048 Dac numerele naturale a i b satisfac
2 2
3 2 b b a a + = + ,
artai c a b este ptrat perfect .
Adriana i Lucian Dragomir , Oelu-Rou
IX.049 Fie f :
*

*
o funcie cu proprietatea c
) ( ) ( ) ( n f m f n m f + + , m , n
*
.
( a ) Demonstrai c : ) ( ) (
1
... ) 2 (
2
1
) 1 ( n f n f
n
f f + + + , n
( b ) Determinai funciile f :
*

*
pentru care avem egalitate n
inegalitatea de la ( a ) .
Lucian Dragomir , Oelu-Rou
IX.050 Se consider o funcie f : care are proprietile :
a ) ) ( ) ( ) ( y f x f xy f + = , x , y ;
b ) 0 ) ( = n f pentru orice n care are suma cifrelor egal cu 10 .
Calculai f ( 2006 ) . Lucian Dragomir , Oelu-Rou
72
IX.051 Determinai numerele ntregi x , y , z , t care satisfac :

= + +
= + +
3 2 2 2
2
t z y x
t z y x

Lucian Dragomir , Oelu-Rou
Clasa a X-a
X,044 Dac a , b , c , d ( 0 , ) , s se demonstreze c :

d c b a
abc
d
dab
c
cda
b
bcd
a
+ + + + + +
4 4 4 4

Prof. dr. Dorin Mrghidanu, Corabia
X.045 Rezolvai ecuaia : 3 2 2 2
} { ] [
= + +
x x x

Prof. Felix Arhire , Galai
X.046 a ) Stabilii care dintre urmtoarele numere este mai mare :
6 log
9
= a sau 8 log
12
= b ;
b ) Determinai perechile ( x , y ) de numere ntregi care satisfac :

=
+
= +
2 2
2 2
2 2
2
log log
x y
y x
y x
y x

Prof. Lucian Dragomir , Oelu-Rou
X.047.Rezolvai:
2 3 *
( 1) ( 1)( 2)
7 7 ... 7 7 , n ,n 2
2 2
x x nx x
n n n n +
+ + + + = N ,.
Prof. Nicolae Dragomir , Prof. Tudor Deaconu , Reia
X.048 Se consider un cerc C de centru O i ABC un triunghi oarecare
n planul cercului , iar D este mijlocul laturii ( BC ) .
a ) Determinai un punct M ( AD ) pentru care suma

2 2 2
MC MB MA S + + = este minim ;
b ) Determinai un punct N C pentru care suma

2 2 2
NC NB NA T + + = este minim .
Prof. Nicolae Stniloiu , Boca
X.049 Numim graf (neorientat) o mulime finit de puncte numite
noduri/vrfuri i mulimea perechilor de puncte (neordonate) [x
i
,xj].
Notm: G = { {x
1,
x
2,,
x
n,},{
[x
i
,xj] i, j 1...n }}
ntr-un graf, un lan este o succesiune de vrfuri (minim dou)
alese astfel nct oricare dou vrfuri succesive definesc o muchie [x
i
,xj],
iar n succesiunea de muchii au fiecare un punct comun.{ [x
i
,x
i+1
], [x
i+1
,x
k
]
[x
k
,x
l
]}
w
w
w
.
n
e
u
t
r
i
n
o
.
r
o
73
Literele cuvntului LAN alctuiesc un graf cu10 vrfuri.Vrfurile
grafului sunt alese dintre vrfurile a trei ptrate alturate , de latura 1 i
mijloace ale segmentelor.
a) Care este suma lungimilor segmentelor din care este format cuvntul
LAN?
b) Cte lanturi se pot defini n graf ?
c) Care este lungimea lanului care trece prin toate punctele grafului?
d) n subgraful de 5 vrfuri care lan are lungimea minim?
Prof. Mircea Iucu, Reia
X.050. Cte perechi ( m ,n ) de numere naturale nenule satisfac :

5 3 2 3 2
x x x x x
n
m
=

, x 0 ?
Prof. Lucian Dragomir , Oelu-Rou
X.051. . Fie
4 3 2 1
, , , z z z z patru numere complexe. S se arate c dac
exist
0
z a..
0 4 0 3 0 2 0 1
z z z z z z z z = = = atunci numrul:
3 2
3 4
1 4
1 2
z z
z z
z z
z z

este real. Prof.Nicolae Stniloiu,Boca


Clasa a XI-a
XI.044 Fie AM
n
3
27
n
( ), A I = C i 0 ) I 3 (A det
n
.Calculai
) I 3 A det(
n
+ . Prof. Viorel Botea, Brila
XI.045 S se rezolve n M
2
(C) ecuaia:

13 7
9 5
X
1 1
1 1
X
Prof.Dan Negulescu,Brila
XI.046 Fie nN* i M mulimea matricelor ptratice de ordinul n,
inversabile n M
n
(R), avnd elementele n mulimea {1, 2, 3, , 2006}.
S se arate c mulimea M are un numr par de elemente.
prof. Marian Baroni , Galai
XI.047 Se consider matricea

=
2 0
2006 2
A . Determinai matricele
X M
2
( ) care satisfac egalitatea : .
2005
A X X = +
Prof. Nicolae Dragomir , Prof. Tudor Deaconu , Reia
XI.048 Fie A,BM
3
() cu det(A)=det(B) =1.Artai c B A C + = 3 e
matrice inversabil . Prof. Antoanela Buzescu , Caransebe
74
XI.049 Calculai
[ ]
[ ] [ ] n n
n n
n
2
2
lim
3
3
+
+


Prof. Antoanela Buzescu , Caransebe
XI.050 Determinai limita irului
1
) (
n n
x definit prin
2
1
> x , 1 2 ) 1 (
2
1
2
+ = +
+ n n n n
x x x x , n , n 1 .
Prof. Lucian Dragomir , Oelu-Rou
XI.051 Fie
2
( ) A M R cu proprietile
2
det( 3 ) 4 A I =
si
2
det( 2 ) 9 A I + = . S se determine
( )
2
n
A I pentru , 2 n n N .
Prof. Nchil Petre,Ploieti
Clasa a XII-a
XII.044 Fie H un subgrup al unui grup G astfel nct G H are 2006
elemente.
a) S se arate c G are cel mult 4012 elemente;
b) S se construiasc un exemplu de subgrup H cu 2006 elemente al
unui grup cu exact 4012 elemente.
prof. Marian Baroni , Galai
XII.045 Fie ( , ) G un grup abelian i a G . Definim pe G operaia
a

astfel: , ,
a
x y x y a x y G = . Fie
{ }
a
H a G =
. S se arate c:
a) ( , )
a
G este un grup abelian;
b) ( , )
a
G si ( , )
b
G sunt grupuri izomorfe , a b G ;
c) pe mulimea H se poate defini o lege de compoziie astfel nct
( , ) H s fie un grup i acest grup s fie izomorf cu grupul ( , ) G .
Prof. Apostolescu Cezar,Ploieti
XII.046 Calculai primitivele funciei
2
2
1
: , ( ) arccos
1
x
f f x
x

=

+

R R
.
Olimpiad Prahova,2006
XII.047 Fie (G, ) un grup finit cu p elemente (pN, prim). S se
demonstreze c dac f :GG este un morfism astfel nct exist
xG-{e}, cu f(x)=x, atunci f=1
G
Olimpiad Vaslui 2006
XII.048 Determinai o funcie : f R R, f(0) = 0, tiind c admite o
primitiv F astfel nct F(x) + f(x) = sinx, oricare ar fi xR.
Olimpiad Vaslui 2006

w
w
w
.
n
e
u
t
r
i
n
o
.
r
o
75
XII.049. Studiai primitivabilitatea funciei f : (-1,1)R,
f(x) =

+
0 , 0
} 0 { \ ) 1 , 1 ( ,
1
arcsin
2
x
x
x
x
arctg x
. Olimpiad Cluj 2006
XII.050 Fie (G, ) un grup cu elementul neutru e.
1) Dac x
2
= e , G x , artai c grupul este comutativ.
2) Dac G este finit, comutativ i x
2
= e pentru mai mult de
jumtate din elementele lui G artai c x
2
= e , G x .
Prof. V. Lupor , ISJ Cluj , Prof.A. Macovei C.N. G. Cobuc Cluj
XII.051 Fie f : [0,1][0,1] o funcie primitivabil i F o primitiv strict
cresctoare a lui f pe [0,1] cu proprietatea c F(0) = 0. Artai c
) 1 , 0 ( c astfel nct F(c) = ) (
2 2
1
c f
c
c

.
Prof. M. Bojescu , L.T. A. Iancu,Cluj-Napoca

Rubrica rezolvitorilor
punctaje realizate pentru soluiile problemelor din RMCS nr. 16
( n parantez apare punctajul total realizat pentru concurs ); cei care
nu apar nu au trimis soluii sau nu au respectat termenul
Clasa a III-a (din 15 septembrie a IV-a):
Liceul Hercules Bile Herculane ( nv. Floarea Kuszai ) :
andru Ilie Daniel 112(185) , Dobreanu Rzvan 103(172) , Lozovanu
Dumitru 134(234) , Croitoru Ioana Sabina -,(95) ; (nv.Doina Zah ) T r k
Bogdan 117(217) , Daniel Coman 125(210) , Mihart Georgiana 126(226) ,
Dancu Anca 126(226) , Dimcea Ana-Maria-Alexandra (100) ,
Gherghina Liviu-Nicu 97(185) , Ferescu Liana-Maria 125(220) ,
Domilescu Manuel-Ilie 118(210) ,Ciopec Oana 123(123)
coala nr. 1 Oravia (nv. Merima Velcot ) : Prvu Ancua Iulia ,(67)
Liceul Pedagogic C.D.Loga Caransebe ( nv. Monica Pelin ,
Monica Urban ) : Lala Timotei (70) , Iordache Andreea-Claudia-,(100) ,
Bzvan Ctlina 87(157) , Bzvan Rzvan 67(137) , Bojin Clina
Milica-, (67)
Liceul Traian Doda Caransebe ( nv. Marinela Galescu ) :
Dragomir Ioana-tefania 80(177) , Mura Ana-Maria (82) , Moraru
Drago (50) , Suru Alexandru Rzvan-( 86) , Coste Anastasia (97) ,
Voicu Vlad (97)
coala General 2 Reia (inst.Ozana Scrin) : Lctuu Georgiana
42(42)
76
Clasa a IV-a(din 15 septembrie a V-a ) :
coala Bnia ( nv. Cristian Pirtea ) : Odobaa Daniel 158(251)
Liceul Pedagogic C.D.Loga Caransebe ( nv. Pelin Monica ,
Monica Urban ) :Leon Natalia-Emilia 60 ,
(inst. Mirela Ttar ) : Panti Antonia 100 , erbescu Andreea 78 , Neuman
Liviu 98 , Cernescu Sebastian 100 , Pop Silvia 115 ,
Liceul Traian Doda Caransebe ( nv. Elena Crstea ) :
Szabo Ildiko 149(244)
coala General 1 Oravia (nv.Liliana Crciun) : Serafin Dennis
George 104(104)
coala General 2 Reia ( inst. Ozana Scrin , nv. Ana Modoran ,
nv. Georgeta Gai , nv.Mrioara Popescu ) :
Btea Flavius (20) , Dieaconu Estera 53(96) , Saec Ion Cosmin 35(61) ,
Blu Lorena (20), Baierle Amalia 65(85) , Izvernar Daniel Otniel
45(72) , Bana Nicolae Alex 37(57) , Barbu Bogdan-( 98) , Ungurean
Simona Roxana -(28) , Wettori Michael Sebastian (100) , Balasan
Roberta Andreea-( 65) , Rogge Petra-Ana 126(226) , eudan Adina-
( 100) , Drghici Livia-Liliana 133(233) , Tomescu Alina-Nicoleta-
( 100) ,Uzoni Ribana-Alexandra (100) , Descu Andrei (100) ,Brebenariu
Octavia-Patricia (100) , Mregea Natalia Patricia (100) , Blaga Georgiana
(100) , Popa Andreea 99(199) , Onofrei Iulia 27(97) , Borchescu Daiana
Maria 37(76) , Zria Gergiana (60) , Bolf Larisa (30) , Aghescu Monica
Elena 134(234) , Darie Mdlina Mihaela (70) , Lungu Cosmin (30) ,
ou Mdlina (88) , Toader Alexandra Anastasia (40) , arga Robert
(80) , Toader Teodora (40) , Nicorici Bogdan (100) , Marin Remus
93(93),Ruja Iulia Maria 118(118),Plie Lorena 54(54),Oanc Maria
Alexandra 54(54).
coala General 9 Reia ( nv. Adina Belu )
Peptan Alexandru-Florin 136(236) , Grdinaru Adelina 98 , Arusoaiei
Iulian 98 , Manciu Bogdan 99 , Nedelea Adrian-Gabriel 95 , Lazr Silviu
Ioan 178(298) , ( inst. Mariana Mitric ) : Muscai Lorena 145(243)
Clasa a V-a (clasa a VI-a din toamn):
coala nr.1 Anina (prof. Manuela Skopecs):
Rotaru Ana-Maria (40) , Drgil Patricia (79) ,Vrnceanu Cezar (35) ,
Srghie Bianca (56)
Liceul Hercules Bile Herculane ( prof.Marius Golopena):
Iacobici Pavelina (30) , Anton Alexandru Lucian (83) ( trimite toate
problemele ntr-un singur plic ! ) , Tabugan Clina Dana ( 139 ) ,
Lolea Sandra (68) , Muic Grozvescu Mihaela (48) ,Popeang Raluca
w
w
w
.
n
e
u
t
r
i
n
o
.
r
o
77
tefania (132) , Basarab George (126) , Martin Patricia (17) ,Mncescu
Maria-Manuela(28) , Cernea Alexandra (15)
coala Bozovici (prof.Iosif Gin ) :
Bratosin Felix (65),Barbe Cezara (55) , Punescu Alexandra (55) ,Bin
Daiana (25) , Bcil Cristiana (67)
coala nr.2 Caransebe ( prof. Corci Carina ):
Agape Oana Gabriela 180(354) , Bdlu Alexandru 57(153) ,
Brbuceanu Florina (122) , Margan Anua Roxana 144(144),Dumitracu
Andreea 147(295).
Liceul Pedagogic C.D.Loga Caransebe ( prof.Maria Mirulescu)
Sseac Iulia Irina 154(231) , Ttar Octavian 147(282) , Ion Rzvan
(120) , Vela Silviu 65(131) , Antonescu Ionica Nicoleta (38) , Timofte
Tina 129(204)
Liceul Traian Doda Caransebe (prof.Adrian Dragomir ):
Stoicnescu Gelu (165) , Rada Cristiana 143(300) , Keleti Edith 143(286) ,
Stepanescu Mihai (80) , Burciu Daniel (36) , Puchi Daniel (80)
coala nr.1 Moldova-Nou ( prof. Marioara Radosavlevici )
Grjan Laura Nicoleta (74) , Craiovan Andreia Dana (77)
coala Ciclova Romn (prof. Geta Mcoi )Bnuc Vasilic Angel (40)
coala General 2 Reia (prof.Mariana Drghici):
Meter Amalia (87) , Mihil Flavius (41) , Mo Daria (46) , Cernea
Serena 164(221) , Scutaru Lavinia (15) , Radcu Antonia (50) , Moldovanu
Alina (33) , Irina Ciorogar 185(252) , Pascu Andra Diana 214(333),Florea
Niki-Alexandru 167(167).
coala nr. 1 Oravia ( prof. Camelia Prvu ) : Pelian-Popa Ioana
110(139)
Grup colar Industrial Oelu-Rou ( prof.Adriana Dragomir) :
Dumitresc Cecilia Graiela (143), Albai Cosmin (72) , Dragomir Claudiu
(63) , Nasta Laura ( 145) , Gemnariu Trienic (56) .
coala General 1 Oelu-Rou (prof.Amalia Popa):Bu Anamaria
54(54)
coala General nr.3 Oelu-Rou (prof.Boldea Felicia ) :
Buzuriu George (113)
coala Rusca-Teregova (prof.Ciuc Sorin ) :
Stepanescu Georgeta Mihaela 23(41), Codopan Florinela 42(146),Milu
Ionela 28(47),Banda Traian Dani 75, Humia Maria 42(86) ,Blaj Marinela
Alisa 44(140) , Berzescu Nicolae (22) , Davidescu Toma (38) , Curmei
Roxana Andreea( 24)
78
coala Vrciorova ( prof. Ioan Liuba ) : Mran Marius 57(165),Gapar
Nicolae 136(136).

Clasa a VI-a(a VII-a din 15 septembrie)
Grup colar Anina ( prof. Petrior Neagoe ) : Grjan Oana Nicoleta (10) ,
Radu Bianca 50(60).
coala nr. 1 Anina ( prof. Marin Constantin Cleiu ) :
Borcean Andreas (38) , Juraszic Claudia (40) .
coala Bozovici ( prof. Maria Bololoi) : Borchescu Ana Maria 52(122) ,
Petre Estera Alina (70) , Borozan Florina Elisaveta 34(123).
Fr a meniona clasa : Hehea Adina Elena 51(51) ( n-am descifrat
numele bine )
Liceul Traian Doda Caransebe (prof.Delia Dragomir):
Szabo Cristian 218(287) , Mocanu Ioana Dora 204(281), Peia Vigia
Alexandra (50)
Liceul Pedagogic C.D.Loga Caransebe(prof.Dorina Humia):
Pleko Cosmin Peter (28) , Semenescu Anca 213(373) ,Borcean
Gheorghe (63) ,Bob Cristiana (57) ; (prof. Maria Mirulescu), Todorovici
Lucian (45) , Vladu Cristian (76) , Matei Sergiu (30),Antonescu Ionica
Nicoleta 14(14)
coala nr.2 Caransebe ( prof. Corci Carina ): Antoce Alexandru
125(222)
Colegiul Naional Carol I Craiova ( prof. Monica Stanca ) Stanciu Ioan
(113)
Liceul de Art Reia (prof.Adriana Mara ) :Goicovici Denisa (72)
coala General 9Reia ( prof. Irina Avramescu ) :
Kormos Nicholas 47(80).
coala Rusca-Teregova (prof.Ciuc Sorin ):
Paan Petru 100(226) , Linu Florin Cosmin (64) ,Blaj Ilie Dnu (68) ,
Vernicua Petronela (63) , Stepanescu Elisabeta (34) , Banda Vasile (36) ,
Banda Ionela Mitra (43) , Dumitric Eva Daniela (31) , Stancu Ana
Maria (9) , Berzescu Maria (32) , Humia Ana (15) , Stan Stana ( 8 )
coala General nr. 1 Oelu-Rou ( prof.Heidi Feil , Cecon Iulia ):
Duma Andrei (133) , Bistrian Florina (55) , Ivu Nicoleta (65) .

Clasa a VII-a(a VIII-a din 15 septembrie)
coala nr.1 Anina (prof. Manuela Skopecs):
Golmba Pavelina-Adelina (20) , Cleiu Marian Ctlin (60) , Tatar
Santra Sorina(16)
w
w
w
.
n
e
u
t
r
i
n
o
.
r
o
79
coala nr.2Anina ( prof. Nicolae Seracin ) : Busa Bianca(10)
coala Rusca-Teregova (prof.Ciuc Sorin ) :Stepanescu Ana Patricia
32(32), Humia Toma (29) , Iciu Gheorghe (35) , Stepanescu Mihai (27) ,
Raduia tefan (31) , Gherga Ionu-Barbu (40) , Banda Anca (23) ,
Stepanescu Ian (32) , Moac Ion(16) , Rdoi Georgeta (42) , Gherga
Petru (29) , Popa Petru-Ionu (42) , Banda Iosif (21) ,Dumitric Octavian
(17) ; (prof.Ilie Damian ) : Ciuc Cristian Sorin 80(148)
Liceul Hercules Bile Herculane ( prof. Constantin Bolbotin ) :
Pleanu Oana Georgiana (10), Dimcea Ion-Cristian (97)
coala Berzovia ( prof. Dan Miholcea ) : Chisli Alexandra 35(75)
Liceul Traian Doda Caransebe (prof.Delia Dragomir):
Novcescu Dorin 76(161) , Zanfir Cristian 148(208),Vid Cristina(70) ,
Baneu Petru 65(145),Galescu Dan 63(63)
Liceul Pedagogic C.D.Loga Caransebe(prof.Diana Hurduzeu ) :
Prunar Victor 334(544).
coala General 2 Reia (prof.Marius andru):
Meter Sergiu (30)
Liceul de Art Reia (prof.Adriana Mara ) : Cherloab Edith (77)
coala General nr. 1 Oelu-Rou (Prof. Heidi Feil ) : Atinge Carina
(107) , Cococeanu Oana Maria (125)
coala General nr. 3 Oelu-Rou ( prof.Felicia Boldea ):
tefnig Sebastian (65) , Lazr Dinu (58) , Silianovici Alin (46) ,
Brng Sergiu (69)

Clasa a VIII-a(adic prima clas de liceu de la toamn)
coala Dalboe (prof.Pavel Rncu) : Piigane Elena Diana 110(110)
coala Rusca-Teregova (prof.Ciuc Sorin) : Codopan George (48) ,
Humia Maria Mirabela(26) , Gherga Patricia (26) , Stepanescu Anca-
Liliana (25),Stepanescu Adamescu Ioan (32) , Humia Elisabeta (33) ,
Cobel tefania Ionela (25) , Gherga Constantin (30) , Banda Ioan (12) ,
Banda Maria (25) , Gherga-Blaj Elisabeta-Ionica (24).
coala General nr. 2 Boca (prof. Veronica Todor ) :
Stniloiu Ovidiu 130(285).
Liceul Pedagogic C.D.Loga Caransebe(prof.Lavinia Moatr ):
Milcu Roxana 210(332) , Timofte Andrei 163(231) , Cristescu Loga Cella
(60) , Moatr Alexandra 131(211) , Vlad Adina 215(404) , Megan Ligia
56(111) , Plotinaru Diana 40
coala General nr. 3 Oelu-Rou ( prof.Felicia Boldea ):
Lupu Vlad 107 (107)
80
Clasa a IX-a( a X-a din 15 septembrie)
Liceul Hercules Bile Herculane ( prof.Marius Golopena):
Fenean Manuela 55(121) , Caraiman Gabriela Sofica 40(106).
Liceul Teoretic Eftimie Murgu Bozovici (prof.George Pascariu ):
uvei Pavel (9 ).
Liceul Pedagogic C.D.Loga Caransebe(prof.Lavinia Moatr ):
Kremer Emanuela (83) , Gurgu Caius 39(103) , Iliescu Marcel (28) .
Liceul General Dragalina Oravia (prof. Mihai Lazarov ) :
Nezbeda Harald (42) , Rinariu Lucian 53(133)
Grup colar Industrial Oelu-Rou ( prof.Lucian Dragomir ):
Ungura Drago 115(221), Dragomir Lucia 80(125) , Buzuriu Alina
50(102) , Popa Roxana 50(95) , Muntean Cristian 50(95) .

Clasa a X-a(a XI-a din toamn)
Liceul Pedagogic C.D.Loga Caransebe ( prof.Maria Mirulescu):
Labo Laureniu 109(154),Roat Ramona (21) , Mrgan Larisa (23) ,
Munteanu Laura Loredana (35) , Stnilescu Maria (29) ,
Colan Anca (26) , Bmescu Monica (23) , Cornean Cristian(25) , Beja
Ancua (36) , Ciortan Oana (35) , Ionescu Alin (25)
Liceul Traian Doda Caransebe (prof.Lavinia Moatr ) :
Zoican Andrei (23) , Voinea Alexandra 54(129) , Crba Florentina
Angela (68) , Dochin Luminia 50(118) , Mutuleanu Alexandra 21(78) ,
Cuitoi Simina 8(104) , Petru Laura 57(157) , Aghescu Loredana (73) ,
Guulescu Oana 22(113) , Burghelea Bogdan (43) , Piele Ionu 22(70)
( prof. Delia Dragomir ) : Beldie Anca 157(215) , Iacob Alexandra
56(126) , Fril Alina-Alexandra 47(120).
Liceul Traian Lalescu Reia (prof.Ovidiu Bdescu) : Popovici Doru
Adrian Thom 167(167)
Grup colar Industrial Oelu-Rou ( prof.Lucian Dragomir ):
Istodor Cosmin 106(160) , Ciobanu Constantin,zis Costel (42).

Clasa a XI-a(a XII-a din toamn)
Liceul Teoretic Traian Doda Caransebe (prof.Lavinia Moatr):Enache
Bianca Emilia (36) , Murean Viorel Dan (40) , Gherghinu Florin (40)

You might also like